Sie sind auf Seite 1von 36

P1: Trim: 8.375in × 10.875in Top: 0.373in Gutter: 0.

664in
LWBK915-23 LWW-KodaKimble-educational October 27, 2011 8:22

SECTION 3: PULMONARY DISORDERS

Asthma 23
Timothy H. Self, Cary R. Chrisman, and Christopher K. Finch

C O R E P R I N C I P L E S
CHAPTER CASES

1 Asthma is a chronic inflammatory disorder of the airways in which many cells and Case 23-1 (Questions 1, 2),
cellular elements play a role. Airway inflammation also causes an increase in the Case 23-3 (Question 1),
existing bronchial hyperresponsiveness to a variety of stimuli. Bronchospasm is Case 23-5 (Question 6),
another key feature of asthma. Case 23-14 (Question 1)

2 The clinical presentation of asthma includes recurrent episodes of wheezing, Case 23-1 (Questions 1, 2),
breathlessness, chest tightness, and cough, particularly at night and in the early Case 23-2 (Questions 1, 7, 8),
morning. These episodes are usually associated with widespread but variable Case 23-3 (Question 1),
airflow obstruction that is often reversible either spontaneously or with treatment. Case 23-4 (Question 1),
Case 23-7 (Question 1),
Case 23-13 (Questions 1–3)

3 Long-term management of persistent asthma according to national and Case 23-1 (Questions 4–9),
international guidelines is aimed at reducing airway inflammation. Environmental Case 23-2 (Questions 2–6,
control, inhaled corticosteroids, and management of comorbidities that worsen 9, 10, 12),
asthma are core management principles. Major treatments of acute exacerbations Case 23-3 (Questions 2–4),
include frequent inhaled short-acting β2 -agonists and systemic corticosteroids. Case 23-4 (Question 1),
Case 23-5 (Questions 1–5),
Case 23-6 (Question 1),
Case 23-7 (Questions 1, 2),
Case 23-8 (Question 1),
Case 23-9 (Question 1),
Case 23-10 (Questions 1, 2),
Case 23-11 (Question 1),
Case 23-13 (Question 3),
Case 23-14 (Question 1)

4 Monitoring parameters include spirometric measures such as forced expiratory Case 23-1 (Questions 1–3),
volume in 1 second (FEV1 ). Patient self-monitoring includes peak expiratory flow Case 23-2 (Questions 1, 4,
(PEF) as well as symptom assessment. Acute care monitoring includes FEV1 , PEF, 7, 8, 11)
arterial O2 saturation, and arterial blood gases.

5 Based on current evidence, the number of therapeutic controversies is relatively Case 23-5 (Questions 3, 4),
small. Which patients should receive long-acting inhaled β2 -agonists is one topic of Case 23-16 (Questions 1–4)
some debate.

6 Patient education is essential for optimal asthma management. Teaching patients Case 23-3 (Questions 2, 4),
with persistent asthma about daily use of preventive therapy is critical. Education Case 23-5 (Question 2),
regarding correct use of inhalation devices is frequently not done, but is absolutely Case 23-12 (Question 1),
necessary. Case 23-17 (Question 1)

7 Improving outcomes, including reducing emergency department visits, Case 23-9 (Question 1),
hospitalizations, and unscheduled office visits, and enhancing quality of life are all Case 23-15 (Question 1)
achievable by use of the principles of national guidelines.

565
P1: Trim: 8.375in × 10.875in Top: 0.373in Gutter: 0.664in
LWBK915-23 LWW-KodaKimble-educational October 27, 2011 8:22

566 ASTHMA who have older siblings, early exposure to day care, and typical
childhood infections are more likely to activate TH 1 responses
According to the National Institutes of Health (NIH) Expert Panel (protective immunity), resulting in an appropriate balance of TH 1
Report 3 (EPR-3), Guidelines for the Diagnosis and Management to TH 2 cells and the cytokines that they produce. On the other
of Asthma,1 asthma is defined as a chronic inflammatory disorder hand, if the immune response is predominately from TH 2 cells
of the airways in which many cells and cellular elements play a (which produce cytokines that mediate allergic inflammation),
role, in particular, mast cells, eosinophils, T lymphocytes, neu- development of diseases such as asthma is more likely. Examples
trophils, and epithelial cells. In susceptible persons, this inflam- of factors favoring this imbalance include the common use of
mation causes recurrent episodes of wheezing, breathlessness, antimicrobial agents, urban environment, and Western lifestyle.
chest tightness, and cough, particularly at night and in the early Further insights into the pathogenesis of asthma continue to be
morning. These episodes are usually associated with widespread discovered.1,7,11,12
but variable airflow obstruction that is often reversible either
spontaneously or with treatment. The inflammation also causes Pathophysiology
an increase in the existing bronchial hyperresponsiveness to a
variety of stimuli.2 This definition of asthma is the same as the Asthma is caused by a complex interaction between inflamma-
1997 NIH guidelines2 and has evolved from earlier national and tory cells and mediators. As noted in the definition of asthma,
Section 3

international guidelines.3–6 mast cells, eosinophils, T lymphocytes, neutrophils, and epithe-


At least 22 million Americans have asthma.1 It is an under- lial cells are of central importance. The bronchial epithelium in
diagnosed and undertreated condition that is estimated to have asthmatic patients has been described as fragile, with various
overall costs exceeding $12 billion annually in the United States.7 abnormalities including destruction of ciliated cells and overex-
Asthma is the leading cause of lost school days in children and is pression of epidermal growth factors.13 Figure 23-1 depicts the
complex interaction of cells and mediators associated with air-
Pulmonary Disorders

a common cause of lost workdays among adults.


Mortality from asthma has decreased in the 21st century, from way inflammation.
4,657 deaths in 1999 to 3,447 deaths in 2007 in the United States After exposure to an asthma-precipitating factor (e.g., aeroal-
according to the Centers for Disease Control and Prevention,8,9 lergen), inflammatory mediators are released from bronchial
but morbidity and mortality are still unacceptably high, especially mast cells, macrophages, T lymphocytes, and epithelial cells.
in inner-city minority populations. This chapter emphasizes the These mediators direct the migration and activation of other in-
2007 NIH EPR-3 guidelines.1 Application of the principles of flammatory cells, most notably eosinophils, to the airways.1,11,12
these recent guidelines by clinicians and patients is vital to further Eosinophils release biochemicals (e.g., major basic protein and
reducing asthma morbidity and mortality. eosinophil cationic protein) that cause airway injury, including
epithelial damage, mucus hypersecretion, and increased reactiv-
ity of smooth muscle.1,7,11
Research continues to determine the role of a subpopulation
Etiology of T lymphocytes (TH 2) in asthmatic airway inflammation.1,11
Childhood-onset asthma is usually associated with atopy, which is TH 2 lymphocytes release cytokines (e.g., interleukin [IL] 4 and
the genetic predisposition for the development of immunoglobu- IL-5) that at least partially control the activation and enhanced
lin E (IgE)–mediated response to common aeroallergens. Atopy is survival of eosinophils.1,4,11 The complexity of airway inflamma-
the strongest predisposing factor in the development of asthma.1 tion is indicated by the fact that at least 27 cytokines may have a
A very common presentation of asthma is a child with a positive role in the pathophysiology of asthma.11 In addition, at least 18
family history of asthma and allergy to tree and grass pollen, chemokines (e.g., eotaxins) have been identified that are impor-
house dust mites, household pets, and molds. tant in delivery of eosinophils to the airways.11 One biomarker
Adult-onset asthma may also be associated with atopy, but of airway inflammation is exhaled nitric oxide (NO), which has
many adults with asthma have a negative family history and neg- been used as a treatment guide in chronic asthma.1 Bronchial
ative skin tests to common aeroallergens. Some of these patients NO has been found to be elevated during periods of exacerba-
may have nasal polyps, aspirin sensitivity, and sinusitis. In the tions and is measurably decreased with administration of inhaled
British 1958 birth cohort study, participants were monitored steroids but not β 2 -agonists.1,14 Failure to adequately minimize
for wheezing and asthma at periodic intervals from birth into severe and long-term airway inflammation in asthma may result
their mid-forties.10 In the subset of patients who were seemingly in airway remodeling in some patients. Airway remodeling refers
asymptomatic during late adolescence and early adulthood, the to structural changes, including an alteration in the amount and
presence of asthma at 42 years of age was significantly higher composition of the extracellular matrix in the airway wall, leading
in those patients who had a history of wheezing in childhood. to airflow obstruction that eventually may become only partially
Exposure to factors (e.g., wood dust, chemicals) at the work- reversible.1,15
place that may cause airway inflammation is also important in Hyperreactivity (defined as an exaggerated response of
many adults. Inflammatory mechanisms are similar, but not the bronchial smooth muscles to trigger stimuli) of the airways to
same, as in atopic asthma. Some clinicians may still refer to intrin- physical, chemical, immunologic, and pharmacologic stimuli is
sic asthma when referring to these patients and extrinsic asthma pathognomonic of asthma.2 Examples of these stimuli include
when discussing atopic asthma. inhaled allergens; respiratory viral infection; cold, dry air; smoke;
In addition to atopy and exposure to occupational chemi- other pollutants; and methacholine. Endogenous stimuli that
cal sensitizers being major risk factors for the development of can worsen asthma include poorly controlled rhinitis, sinusitis,
asthma, several contributing factors may increase the susceptibil- and gastroesophageal reflux disease.1 In addition, premenstrual
ity to the development of the disease in predisposed individuals.1,5 asthma has been reported, but the exact hormonal mechanism
These factors include viral infections, small size at birth, diet, is not known.16
exposure to tobacco smoke, and environmental pollutants.1,5 Although patients with allergic rhinitis, chronic bronchitis,
Recent literature has focused on the “hygiene hypothesis,” an and cystic fibrosis also experience bronchial hyperreactivity,
imbalance of TH 2 and TH 1 type T lymphocytes, to explain the these patients do not experience bronchiolar constriction as
marked increase in asthma in westernized countries.1,5,8 Infants severely as do patients with asthma. The degree of bronchial
P1: Trim: 8.375in × 10.875in Top: 0.373in Gutter: 0.664in
LWBK915-23 LWW-KodaKimble-educational October 27, 2011 8:22

Airway 567
Bone marrow
Antigen

FIGURE 23-1 Airway inflammation. Inhaled antigen


Mast cell TH2 cell activates mast cells and TH 2 cells in the airway. They in
Eosinophil turn induce the production of mediators of inflammation
Airway (such as histamine and leukotrienes) and cytokines
injury including interleukin 4 and interleukin 5. Interleukin 5
travels to the bone marrow and causes terminal
Histamine Interleukin 4 differentiation of eosinophils. Circulating eosinophils
Granule proteins
Leukotrienes GM-CSF enter the area of allergic inflammation and begin

Chapter 23
Interleukin 5 Prolonged survival Leukotrienes migrating to the lung by rolling, through interactions
with selectins, and eventually adhering to endothelium
Cytokine through the binding of integrins to members of the
activation immunoglobulin superfamily of adhesion proteins:
Chemokines vascular-cell adhesion molecule 1 (VCAM-1) and
(RANTES, eotaxin, intercellular adhesion molecule 1 (ICAM-1). As the
MCP-1, MIP-1α) Selectin eosinophils enter the matrix of the airway through the
influence of various chemokines and cytokines, their

Asthma
VCAM-1 survival is prolonged by interleukin 4 and
ICAM-1 granulocyte-macrophage colony-stimulating factor
(GM-CSF). On activation, the eosinophil releases
inflammatory mediators, such as leukotrienes and
granule proteins, to injure airway tissues. In addition,
Transmigration
Adhesion eosinophils can generate GM-CSF to prolong and
potentiate their survival and contribution to persistent
Endothelium airway inflammation. MCP-1, monocyte chemotactic
protein; MIP-1α, macrophage inflammatory protein;
Blood RANTES, chemokine ligand 5. Adapted with permission
from Busse WW, Lemanske RF Jr. Asthma. N Engl J Med.
2001;344:350.

hyperreactivity of asthmatic patients correlates with the clini- mediators as described previously. Bronchodilators do not block
cal course of their disease, which is characterized by periods the LAR to allergen challenge; corticosteroids block the LAR but
of remissions and exacerbations. During times of remission, a do not affect the EAR; and cromolyn blocks both.2
more intense stimulus is required to produce bronchospasm than Pathologic changes found at autopsy performed on asth-
during times of increased symptoms. Numerous theories have matic patients include (a) marked hypertrophy and hyperplasia
been proposed to explain the bronchial hyperreactivity found in of the bronchial smooth muscle, (b) mucous gland hypertrophy
asthma, yet none fully explains the phenomenon. Inflammation and excessive mucus secretion, and (c) denuded epithelium and
appears to be the primary process in the pathogenesis of bronchial mucosal edema owing to an exudative inflammatory reaction
hyperreactivity; however, neurogenic imbalances in the airways and inflammatory cell infiltration.1 Hyperinflation of the lungs
also may play a significant role.5 Inflamed airways are hyper- from air trapping with extensive mucous plugging is found at
reactive (i.e., irritable). Hyperreactivity can be measured in the
physician’s office by having the patient inhale small concentra-
tions of nebulized methacholine or histamine or by exercise (e.g.,
treadmill). The concentration of aerosolized methacholine or IAR LAR
histamine that decreases the forced expiratory volume in 1 sec-
FEV1 (L)

ond (FEV1 ) by 20% is referred to as the PD20 or the PC20 (provoca-


tive dose or concentration that decreases the FEV1 by 20%).2 An
indicator of optimal anti-inflammatory therapy is an increase in
the PD20 with time as the airways become less inflamed and Allergen
therefore less hyperreactive. exposure
Another concept related to inflammation is “late-phase” ver-
sus “early-phase” asthma (Fig. 23-2). The inhalation of specific 0 1 3 4 5 6 7 8
allergens in atopic asthmatic patients produces immediate bron-
choconstriction (measured by a drop in peak expiratory flow Time (hours)
[PEF] or FEV1 ) that spontaneously improves in an hour or is
FIGURE 23-2 Typical immediate and late asthmatic responses
reversed easily by inhalation of a β 2 -agonist. Although this early seen after exposure to relevant allergen. Immediate asthmatic
asthmatic response (EAR) is blocked by the preadministration of response (IAR) occurs within minutes, whereas late asthmatic
β 2 -agonists, cromolyn, or theophylline, a second bronchocon- response (LAR) occurs several hours after exposure. Patients may
strictive response often occurs 4 to 12 hours later. This late asth- demonstrate isolated IAR, isolated LAR, or dual responses. FEV1 ,
forced expiratory volume in 1 second. Adapted with permission from
matic response (LAR) often is more severe, more prolonged, and Herfindal ET, Gourley DR, eds. Textbook of Therapeutics Drug and
more difficult to reverse with bronchodilators than is the EAR. Disease Management. 7th ed. Baltimore, MD: Lippincott Williams &
The LAR is associated with the influx of inflammatory cells and Wilkins; 2003.
P1: Trim: 8.375in × 10.875in Top: 0.373in Gutter: 0.664in
LWBK915-23 LWW-KodaKimble-educational October 27, 2011 8:22

568
TA B L E 2 3 - 1
Classifying Asthma Severity in Children 0 to 4 Years of Age

Classifying Severity in Children who are not Currently Taking Long-term Control Medication
Classification of Asthma Severity (Children 0–4 Years of Age)
Persistent
Components of Severity Intermittent Mild Moderate Severe

Impairment Symptoms ≤2 days/wk >2 days/wk Daily Throughout the


but not daily day
Nighttime awakenings 0 1–2×/mo 3–4×/mo >l×/wk
SABA use for symptom control ≤2 days/wk >2 days/wk Daily Several times per
(not prevention of EIB) but not daily day
Interference with normal activity None Minor limitation Some limitation Extremely limited
Risk Exacerbations requiring oral 0–1/y ≥2 exacerbation in 6 months requiring oral corticosteroids or ≥4
systemic corticosteroids wheezing episodes in 1 year lasting >1 day AND risk factors for
Section 3

persistent asthma
Consider severity and interval since last exacerbation.
←−−−−−−−−−−− Frequency and severity may fluctuate with time. −−−−−−−−−−−→
Exacerbations of any severity may occur in patients in any severity category.
Level of severity is determined by both impairment and risk. Assess impairment domain by caregiver’s recall of previous 2–4 weeks. Assign severity to
Pulmonary Disorders

the most severe category in which any feature occurs.


At present, there are inadequate data to correspond frequencies of exacerbations with different levels of asthma severity. For treatment purposes,
patients who had ≥2 exacerbations requiring oral corticosteroids in the past 6 months, or ≥4 wheezing episodes in the past year, and who have risk
factors for persistent asthma may be considered the same as patients who have persistent asthma, even in the absence of impairment levels consistent
with persistent asthma.
Classifying Severity in Patients After Asthma Becomes Well Controlled, by Lowest Level of Treatment Required to Maintain Control
Classification of Asthma Severity
Persistent
Intermittent Mild Moderate Severe
Lowest level of treatment required to maintain control Step 1 Step 2 Step 3 or 4 Step 5 or 6
(See Fig. 23-7 for treatment steps.)

EIB, exercise-induced bronchospasm; SABA, short-acting inhaled β 2 -agonist.


Reprinted from National Institutes of Health. Expert Panel Report 3: Guidelines for the Diagnosis and Management of Asthma. Bethesda, MD: National Heart, Lung, and Blood
Institute; 2007. NIH publication 07-4051.

autopsy in patients who have died of acute asthma attacks, but EPR-3 uses the classifications of intermittent, mild persistent,
these changes also are seen at autopsy in asthmatic patients dying moderate persistent, and severe persistent asthma (Tables 23-1–
of other causes. The bronchial smooth muscle hypertrophy and 23-3). The frequency of symptoms is a key component of asthma
mucus hypersecretion are secondary to the chronic inflamma- classification.1 For example, mild persistent asthma is defined as
tory response.17 symptoms more than two times per week or nocturnal symp-
toms (including early morning chest tightness) more than two
times per month. Many clinicians are unaware that this level of
For an animation describing asthma, go to symptoms is defined as persistent asthma. This classification is
http://thepoint.lww.com/AT10e. of major significance when selecting long-term drug therapy in
that daily use of anti-inflammatory agents is an essential part of
management for persistent asthma.1

Symptoms
The heterogeneity of asthma is reflected best in its clinical presen-
Diagnosis and Monitoring
tation. Classically, patients with asthma present with intermittent HISTORY
episodes of expiratory wheezing, coughing, and dyspnea. Some The diagnosis of asthma is based primarily on a detailed history of
patients, however, experience chest tightness or a chronic cough intermittent symptoms of wheezing, chest tightness, shortness
that is not associated with wheezing. There is a wide spectrum of breath, and coughing. These episodes may be worse seasonally
of disease severity, ranging from patients with occasional, mild (e.g., springtime or late summer and early fall) or in association
bouts of breathlessness to patients who wheeze daily despite con- with exercise. History of nocturnal symptoms with awakening
tinuous high dosages of medication. In addition, the severity of in the early morning is a critical component to assess. In addition,
asthma may be influenced by environmental factors (e.g., specific history of symptoms after exposure to other common triggers
seasonal allergens). Symptoms often are associated with exercise (e.g., cats, perfume, secondhand tobacco smoke) is typical (Table
and sleep (refer to Case 23-11, Case 23-12, and Case 23-14). 23-4). A positive family history and the presence of rhinitis or
Classification of asthma severity is of major importance in atopic dermatitis also are significant. After a careful history is
defining initial long-term treatment. Within three age groups, obtained, skin testing may be useful in identifying triggering
P1: Trim: 8.375in × 10.875in Top: 0.373in Gutter: 0.664in
LWBK915-23 LWW-KodaKimble-educational October 27, 2011 8:22

569
TA B L E 2 3 - 2
Classifying Asthma Severity in Children 5 to 11 Years of Age

Classifying Severity in Children who are not Currently Taking Long-term Control Medication
Classification of Asthma Severity (Children 5–11 Years of Age)
Persistent
Components of Severity Intermittent Mild Moderate Severe

Impairment Symptoms ≤2 days/wk >2 days/wk but not Daily Throughout the day
daily
Nighttime awakenings ≤2×/mo 3–4×/mo >1×/wk but not Often 7×/wk
nightly
SABA use for symptom ≤2 days/wk >2 days/wk but not Daily Several times per day
control (not prevention daily
of EIB)
Interference with normal None Minor limitation Some limitation Extremely limited

Chapter 23
activity
Lung function s Normal FEV1
between
exacerbations
s FEV >80% s FEV1 >80% s FEV1 = 60%–80% s FEV1 <60%
1
predicted predicted predicted predicted
s FEV /FVC >85% s FEV /FVC >80% s FEV /FVC s FEV /FVC <75%

Asthma
1 1 1 1
75%–80%
Risk Exacerbations requiring 0–1 in 1 year (see note) ≥2 in 1 year
oral systemic (see note) −−−−−−−−−−−−−−−−−−−−−−−−−−−−−−−−−−−−−−−→
corticosteroids
Consider severity and interval since last exacerbation. Frequency and
←−−−−−−−− −−−−−−−−→
severity may fluctuate with time for patients in any severity category.
Relative annual risk of exacerbations may be related to FEV1 .
Level of severity is determined by both impairment and risk. Assess impairment domain by patient’s or caregiver’s recall of the previous 2–4 weeks and
spirometry. Assign severity to the most severe category in which any feature occurs.
At present, there are inadequate data to correspond frequencies of exacerbations with different levels of asthma severity. In general, more frequent and
intense exacerbations (e.g., requiring urgent, unscheduled care, hospitalization, or ICU admission indicate greater underlying disease severity. For
treatment purposes, patients who had ≥2 exacerbations requiring oral systemic corticosteroids in the past year may be considered the same as
patients who have persistent asthma, even in the absence of impairment levels consistent with persistent asthma.
Classifying Severity in Patients After Asthma Becomes Well Controlled, by Lowest Level of Treatment Required to Maintain Control
Classification of Asthma Severity
Persistent
Intermittent Mild Moderate Severe
Lowest level of treatment required to Step 1 Step 2 Step 3 or 4 Step 5 or 6
maintain control
(See Fig. 23-8 for treatment steps.)

EIB, exercise-induced bronchospasm; FEV1 , forced expiratory volume in 1 second; FVC, forced vital capacity; ICU, intensive care unit; SABA, short-acting β 2 -agonist.
Reprinted from National Institutes of Health. Expert Panel Report 3: Guidelines for the Diagnosis and Management of Asthma. Bethesda, MD: National Heart, Lung, and Blood
Institute; 2007. NIH publication 07-4051.

allergens, but it is only of supportive value in the diagnosis of expiration. The volume of air left after a normal expiration is the
asthma. functional residual capacity (FRC). Total lung capacity (TLC) is
the VC plus the RV. Patients with obstructive lung disease have
PULMONARY FUNCTION TESTS difficulty with expiration; therefore, they tend to have a decreased
The diagnosis of asthma is based in part on demonstration of VC, an increased RV, and a normal TLC. Classic restrictive lung
reversible airway obstruction. A brief discussion of tests to detect diseases (e.g., sarcoidosis, idiopathic pulmonary fibrosis) present
reversibility of airway obstruction is important. Furthermore, a with decrements in all lung volumes.18 Patients also may have
short summary of arterial blood gases (ABGs) is pertinent here mixed lesion diseases, in which case the classic findings are not
in assessing the severity of asthma exacerbations. apparent until the disease has advanced considerably.
The spirometer also can be used to evaluate the performance
SPIROMETRY of the patient’s lungs, thorax, and respiratory muscles in mov-
Lung volumes often are measured to obtain information about ing air into and out of the lungs. Forced expiratory maneuvers
the size of the patient’s lungs because pulmonary diseases can amplify the ventilation abnormalities produced. The single most
affect the volume of air that can be inhaled and exhaled. The tidal useful test for ventilatory dysfunction is the forced expiratory vol-
volume is the volume of air inspired or expired during normal ume (FEV). The FEV is measured by having the patient exhale
breathing. The volume of air blown off after maximal inspiration into the spirometer as forcefully and completely as possible
to full expiration is defined as the vital capacity (VC). The residual after maximal inspiration. The resulting volume curve is plot-
volume (RV) is the volume of air left in the lung after maximal ted against time (Fig. 23-3) so that expiratory flow can be
P1: Trim: 8.375in × 10.875in Top: 0.373in Gutter: 0.664in
LWBK915-23 LWW-KodaKimble-educational October 27, 2011 8:22

570
TA B L E 2 3 - 3
Classifying Asthma Severity in Youths ≥12 Years of Age and Adults

Classifying Severity in Patients who are not Currently Taking Long-term Control Medication
Classification of Asthma Severity (Youths ≥12 Years of Age and Adults)
Persistent
Components of Severity Intermittent Mild Moderate Severe

Impairment Symptoms ≤2 days/wk >2 days/wk but not Daily Throughout the
daily day
Nighttime awakenings ≤2×/mo 3–4×/mo >1×/wk but not Often 7×/wk
nightly
SABA use for symptom ≤2 days/wk >2 days/wk but not Daily Several times per
control (not prevention of >1×/day day
EIB)
Normal FEV1/FVC:
Section 3

8–19 years, 85% Interference with normal None Minor limitation Some limitation Extremely limited
20–39 years, 80% activity
40–59 years, 75%
60–80 years, 70%
Lung function s Normal FEV1
between
Pulmonary Disorders

exacerbations
s FEV >80% s FEV1 ≥80% s FEV1 >60% but s FEV1 <60%
1
predicted predicted <80% predicted predicted
s FEV /FVC normal s FEV /FVC normal s FEV /FVC s FEV /FVC
1 1 1 1
reduced 5% reduced >5%
Risk Exacerbations requiring oral ≥2 in 1 year
0–1 in 1 year (see note)
systemic corticosteroids (see note)−−−−−−−−−−−−−−−−−−−−−−−−−−−−−−−−−−−−−→
Consider severity and interval since last exacerbation. Frequency and
←−−−−−− −−−−−−→
severity may fluctuate with time for patients in any severity category.
Relative annual risk of exacerbations may be related to FEV1 .
Level of severity is determined by assessment of both impairment and risk. Assess impairment domain by patient’s or caregiver’s recall of previous
2–4 weeks and spirometry. Assign severity to the most severe category in which any feature occurs.
At present, there are inadequate data to correspond frequencies of exacerbations with different levels of asthma severity. In general, more frequent and
intense exacerbations (e.g., requiring urgent, unscheduled care, hospitalization, or ICU admission) indicate greater underlying disease severity. For
treatment purposes, patients who had ≥2 exacerbations requiring oral systemic corticosteroids in the past year may be considered the same as
patients who have persistent asthma, even in the absence of impairment levels consistent with persistent asthma.
Classifying Severity in Patients After Asthma Becomes Well Controlled, by Lowest Level of Treatment Required to Maintain Control
Classification of Asthma Severity
Persistent
Intermittent Mild Moderate Severe
Lowest level of treatment required to maintain Step 1 Step 2 Step 3 or 4 Step 5 or 6
control
(See Fig. 23-9 for treatment steps.)
a
EIB, exercise-induced bronchospasm; FEV1 , forced expiratory volume in 1 second: FVC, forced vital capacity; ICU, intensive care unit; SABA, short-acting β 2 -agonist.
Reprinted from National Institutes of Health. Expert Panel Report 3: Guidelines for the Diagnosis and Management of Asthma. Bethesda, MD: National Heart, Lung, and Blood
Institute; 2007. NIH publication 07-4051.

estimated. For a video that shows how to take a lung function onds. Thus, the FEV1 normally is 80% of the FVC. The patient’s
test, see http://www.european-lung-foundation.org/index. breathing ability is compared against “predicted normal” values
php?id=15411. for patients with similar physiologic characteristics because lung
Standard spirometers contain pneumotachographs in the volumes depend on age, race, sex, height, and weight. For exam-
mouthpieces that can measure airflow directly. A number of ple, an average-sized young adult man may have an FVC of 4 to
important measures of lung function are made from the resulting 5 L and a corresponding FEV1 of 3.2 to 4 L. The FEV1 and the
flow–volume curves (Fig. 23-4). The advantages of this technique FVC are the most reproducible of the pulmonary function tests.
include a display of simultaneous flows at any lung volume, visual
estimation of patient effort and cooperation, high reproducibil- PEAK EXPIRATORY FLOW
ity within as well as across individuals, and an analysis of the The PEF is the maximal flow that can be produced during the
distribution of flow limitation.18,19 The FEV1 of the forced vital forced expiration. The PEF can be measured easily with vari-
capacity (FVC, the maximal volume of air exhaled with maxi- ous handheld peak flow meters and commonly is used in emer-
mally forced effort from a position of maximal inspiration) com- gency departments (EDs) and clinics to quickly and objectively
monly is measured to determine the dynamic performance of assess the effectiveness of bronchodilators in the treatment of
the lung in moving air. The FEV1 usually is expressed as a per- acute asthma attacks. Peak flow meters also can be used at
centage of the total volume of air exhaled and is reported as the home by patients with asthma to assess chronic therapy. The
FEV1 to FVC ratio. Healthy persons generally can exhale at least changes in PEF generally parallel those of the FEV1 ; however,
75% to 80% of their VC in 1 second and almost all of it in 3 sec- the PEF is a less reproducible measure than the FEV1 .5 A healthy,
P1: Trim: 8.375in × 10.875in Top: 0.373in Gutter: 0.664in
LWBK915-23 LWW-KodaKimble-educational October 27, 2011 8:22

1 second 571
TA B L E 2 3 - 4
Sample Questions for the Diagnosis and Initial Assessment
of Asthmaa

A “yes” answer to any question suggests that an asthma diagnosis is


likely.
In the past 12 months . . .
s
Have you had a sudden severe episode or recurrent episodes of
coughing, wheezing (high-pitched whistling sounds when breathing
out), chest tightness, or shortness of breath?
s
Have you had colds that “go to the chest” or take more than 10 days
to get over?
s
Have you had coughing, wheezing, or shortness of breath during a
particular season or time of the year?
s
Have you had coughing, wheezing, or shortness of breath in certain
places or when exposed to certain things (e.g., animals, tobacco

Chapter 23
smoke, perfumes)?
s
Have you used any medications that help you breathe better? How
often?
s
Are your symptoms relieved when the medications are used?
In the past 4 weeks, have you had coughing, wheezing, or
shortness of breath . . . FIGURE 23-3 Volume–time curve from a forced expiratory
s
At night that has awakened you? maneuver. FEV1 , forced expiratory volume in 1 second; FVC, forced

Asthma
s
On awakening? vital capacity.
s
After running, moderate exercise, or other physical activity?
a
These questions are examples and do not represent a standardized assessment
or diagnostic instrument. The validity and reliability of these questions have not In simplest terms, restrictive disease limits airflow during inspi-
been assessed. ration, and obstructive disease limits airflow during expiration.
Reprinted from National Institutes of Health. Expert Panel Report 3: Guidelines for Restrictive disease results from a loss of elasticity (e.g., fibrosis,
the Diagnosis and Management of Asthma. Bethesda, MD: National Heart, Lung,
pneumonia) or physical deformities of the chest (e.g., kyphosco-
and Blood Institute; 2007. NIH publication 07-4051.
liosis), with a consequent inability to expand the lung and a
reduced TLC. Therefore, a typical flow–volume curve (Fig.
average-sized young adult man typically has a PEF of 550 to 23-4C) for a patient with restrictive disease shows markedly
700 L/minute. Commercial peak flow meters come with a chart depressed volumes with increased flow rates (when corrected
for patients to determine their predicted normal PEFs based on for the volume).
their sex, age, and height. Whereas restrictive airway diseases limit lung expansion,
obstructive airway diseases (e.g., bronchitis, asthma) narrow air
OBSTRUCTIVE VERSUS RESTRICTIVE passages, create air turbulence, and increase resistance to airflow.
AIRWAY DISEASE In obstructive diseases, maximal expiration may begin at higher-
Generally, pulmonary disorders fall into two categories: those than-normal lung volumes, and the expiratory flow is depressed
that restrict the lungs and thorax and those that obstruct them. (Fig. 23-4B). Resistance to flow is increased at lower lung
Flow (L/s)

1 second

FIGURE 23-4 Flow–volume curves resulting from a forced expiratory maneuver. A: Normal flow–volume curve. B: Typical
pattern for obstructive disease. C: Typical pattern for restrictive disease. FEF 25%, 50%, 75%, forced expiratory flow at 25%,
50%, and 75% of FVC; FEV1 , forced expiratory volume in 1 second; FVC, forced vital capacity; PEFR, peak expiratory flow rate;
RV, residual volume; TLC, total lung capacity.
P1: Trim: 8.375in × 10.875in Top: 0.373in Gutter: 0.664in
LWBK915-23 LWW-KodaKimble-educational October 27, 2011 8:22

572
9
5 Postbronchodilator
8
Postbronchodilator
7 Pre-FEV1 2.71 L
4
Post-FEV1 3.07 L
6
Volume (L)

(13% increase)

Flow (L/s)
3 5
Prebronchodilator
Pre-FEV1 2.71 L 4
Prebronchodilator
2 Post-FEV1 3.07 L (13% increase) 3
2
1
1

0 0
0 1 2 3 4 5 6 7 8 9 10 0 1 2 3 4 5
Time (seconds) Volume (L)
Section 3

FIGURE 23-5 Interpretation of results of spirometry. The graphs depicted are for illustration only. The
interpretation of flow rates may vary with the age of the patient. FEV1 , forced expiratory volume in 1 second.
Adapted from National Institutes of Health. Expert Panel Report 2. Guidelines for the Diagnosis and Management of
Asthma. Bethesda, MD: National Heart, Lung, and Blood Institute; 1997. NIH publication 97-4051.
Pulmonary Disorders

volumes, giving the characteristic scooped-out appearance of or drugs that are known to produce bronchospasm in asthmatic
the obstructive flow–volume curve (Fig. 23-4B). patients (e.g., aerosolized methacholine).

REVERSIBLE AIRWAY OBSTRUCTION BLOOD GAS MEASUREMENTS


Spirometry often is used to determine the reversibility of air- The best indicators of overall lung function (ventilation and dif-
way disease. Although many generally associate reversibility fusion) are the ABGs (i.e., arterial partial pressure of oxygen
with bronchospasm, therapy can improve airflow by reversing [Pao2 ], arterial partial pressure of carbon dioxide [Paco2 ], and
any of the causative pathologic processes of asthma described pH). Although ABG measurements also are dependent on the
previously. Significant clinical reversibility produced from bron- patient’s cardiovascular status, they are indispensable in assess-
chodilators is determined by the tests outlined in Figure 23-5. ing both acute and chronic changes in pulmonary patients. (See
The FEV1 is considered the gold standard test for determining Chapter 9, Acid–Base Disorders, for a review of ABGs.) Another
reversibility of airway disease and bronchodilator efficacy. Sig- means of assessing the patient’s ability to oxygenate tissues ade-
nificant clinical reversibility is defined as a 12% improvement quately is to measure oxygen saturation, which is described by
in FEV1 after administration of a short-acting bronchodilator.1 the following equation:
An improvement of 20% in FEV1 provides noticeable subjective
relief of respiratory symptoms in most patients. For patients with Quantity of O2 actually
a very low baseline FEV1 (e.g., <1 L), an absolute improvement bound to hemoglobin
O2 saturation = × 100
of 250 mL sometimes is considered a better indicator of therapeu- Quantity of O2 that can
tic benefit than assessing percentage of change. In either case, the be bound to hemoglobin (Eq. 23-1)
patient’s subjective clinical impression also should be considered
According to this equation, oxygen saturation is the ratio
when using pulmonary function testing and drug challenges as
between the actual amount of oxygen bound to hemoglobin
predictors for future therapy.
and the potential amount of oxygen that could be bound to
hemoglobin at a given pressure. The denominator in the preced-
LIMITATIONS OF SPIROMETRY ing equation is the oxygen capacity. The normal oxygen satura-
Because the FEV1 and the PEF are both highly effort depen- tion of arterial blood at a Pao2 of 100 mm Hg is 97.5%; that of
dent, complete patient cooperation is required for reliable results. mixed venous blood at a Po2 of 40 mm Hg is about 75%.18
Therefore, spirometric tests often are unobtainable in patients Oxygen saturations can be measured continuously with
who are severely ill as well as in patients who are very old or very transcutaneous monitors. This type of monitoring (pulse oxime-
young. The FEV1 and PEF also are relatively insensitive to small try) is extremely helpful in determining whether supplemen-
airway changes and are therefore unable to detect early mucous tal oxygen therapy is indicated in patients with various chronic
plugging and inflammation in small bronchioles. Although the respiratory diseases. At a Pao2 of less than 60 mm Hg, oxygen
forced expiratory flow measured between 25% and 75% of FVC saturation begins to drop precipitously (Fig. 23-6).
(FEF25%–75% ) is a more sensitive test of small airway obstruction,
it is also much more variable, requiring larger changes (30%–
40%) to be clinically significant. Goals of Therapy
Spirometric pulmonary function tests before and after admin- The EPR-31 established the following goals of therapy to achieve
istration of an inhaled bronchodilator can be useful in assessing control of asthma:
the reversibility of airway obstruction. If significantly depressed
pulmonary function tests are not reversed by the administration Reduce Impairment: (a) Prevent chronic and troublesome
of a bronchodilator acutely, a 2- to 3-week trial of oral cortico- symptoms (e.g., coughing or breathlessness in the night, in
steroid treatment followed by retesting might detect the early morning, or after exertion); (b) maintain (near) “nor-
reversibility.1 mal” pulmonary function; (c) maintain normal activity levels
If pulmonary function is normal or near normal at the time of (including exercise, other physical activities, and attendance
spirometric assessment, the patient can be challenged by exercise at work or school); (d) require infrequent use of short-acting
P1: Trim: 8.375in × 10.875in Top: 0.373in Gutter: 0.664in
LWBK915-23 LWW-KodaKimble-educational October 27, 2011 8:22

ACUTE ASTHMA 573

Assessment
SIGNS AND SYMPTOMS
CASE 23-1
QUESTION 1: Q.C., a 6-year-old, 20-kg girl, presents to the
ED with complaints of dyspnea and coughing that have pro-
gressively worsened during the past 2 days. These symp-
toms were preceded by 3 days of symptoms of a viral
upper respiratory tract infection (sore throat, rhinorrhea,
and coughing). She has experienced several bouts of bron-
chitis in the last 2 years and was hospitalized for pneu-
monia 3 months ago. Q.C. is not being treated with any

Chapter 23
medications at present. Physical examination reveals an
anxious-appearing young girl in moderate respiratory dis-
Po2 (mm Hg) tress with audible expiratory wheezes; occasional coughing;
a prolonged expiratory phase; a hyperinflated chest; and
FIGURE 23-6 The oxygen dissociation curve reveals that the suprasternal, supraclavicular, and intercostal retractions.
percent saturation of hemoglobin increases almost linearly with Bilateral inspiratory and expiratory wheezes with decreased
increases in the arterial O2 tension until a partial pressure of

Asthma
breath sounds on the left side are heard on auscultation.
arterial oxygen (PaO2 ) of 55 to 65 mm Hg is reached. At PaO2
values above this, the increase in hemoglobin saturation becomes Q.C.’s vital signs are as follows: respiratory rate (RR), 30
proportionately less, and relatively little additional oxygen is added breaths/minute; blood pressure (BP), 110/83 mm Hg; heart
to the hemoglobin despite large increases in PaO2 . Adapted with rate, 130 beats/minute; temperature, 37.8◦ C; and pulsus
permission from Guenther CA, Welch MH. Pulmonary Medicine. 2nd paradoxus, 18 mm Hg. Her arterial oxygen saturation (SaO2 )
ed. Philadelphia, PA: JB Lippincott; 1982.
by pulse oximetry is 90%. Q.C. is given O2 to maintain
SaO2 greater than 90% and 2.5 mg of albuterol by neb-
ulizer every 20 minutes for three doses. After the initial
inhaled β 2 -agonists ([SABAs], ≤2 days a week for quick relief treatment, Q.C. claims some subjective improvement and
of symptoms); and (e) meet patients’ and families’ expecta- appears to be more comfortable; however, wheezing on
tions of and satisfaction with asthma care. auscultation becomes louder. What signs and symptoms in
Reduce Risk: (a) Prevent recurrent exacerbations of asthma Q.C. are consistent with acute bronchial obstruction? Does
and minimize the need for ED visits or hospitalizations; increased wheezing after albuterol indicate failure of the
(b) prevent progressive loss of lung function—for children, medication?
prevent reduced lung growth; and (c) provide optimal phar-
macotherapy with minimal or no adverse effects. Asthma is an obstructive lung disease; therefore, the pri-
mary limitation to airflow occurs during expiration. This outflow
obstruction leads to the classic findings of dyspnea, expiratory
MAJOR COMPONENTS OF wheezes, and a prolonged expiratory phase during the ventila-
LONG-TERM MANAGEMENT tory cycle.1 Wheezing is a whistling sound produced by turbu-
To achieve these goals of therapy, EPR-31 also outlines some gen- lent airflow through a constricted opening and usually is more
eral treatment principles. Asthma management has four major prominent on expiration. Thus, the audible expiratory wheezing
components, including (a) measures of asthma assessment and in Q.C. is compatible with bronchial obstruction. In fact, Q.C.’s
monitoring, (b) education for a partnership in asthma care, obstruction is so severe that even inspiratory wheezes and
(c) control of environmental factors and comorbid conditions decreased air movement were detected on auscultation. It is
that affect asthma, and (d) medications. Optimal long-term man- important to realize that the classic symptom of wheezing
agement requires a continuous care approach, including each requires turbulent airflow; therefore, effective therapy of acute
of these four major components, to prevent exacerbations and asthma actually may result in increased wheezing initially as air-
decrease airway inflammation. Early therapeutic interventions flow increases throughout the lung. As a result, Q.C.’s increased
in managing acute exacerbations are very important in decreas- wheezing on auscultation is compatible with her clinical improve-
ing the chance of severe narrowing of the airways. Achieving ment after the albuterol nebulizer treatments.
the goals of asthma therapy also involves individualizing each The coughing experienced by Q.C. is another common find-
patient’s therapy. In addition, optimal care involves establishing ing associated with acute asthma attacks. The coughing may
a “partnership” between the patient, the patient’s family, and the caused by stimulation of “irritant receptors” in the bronchi by
clinician. the chemical mediators of inflammation (e.g., leukotrienes) that
For most patients with asthma, the condition can be well are released from mast cells or from the mechanics of smooth
controlled by using the step–care approach recommended by muscle contraction.
EPR-31 (Figs. 23-7, 23-8, 23-9). A concerted effort in patient edu- In the progression of an asthma attack, the small airways
cation as an integral part of state-of-the-art long-term manage- become completely occluded during expiration, and air can
ment has been demonstrated to improve outcomes, including be trapped behind the occlusion; therefore, the patient has to
quality of life in patients with asthma. Because of the excellent breathe at higher-than-normal lung volumes.1 Consequently, the
outcomes associated with optimal long-term management, if a thoracic cavity becomes hyperexpanded, and the diaphragm is
patient requires an ED visit or hospitalization, great care should lowered. As a result, the patient must use the accessory mus-
be given to determining how the acute-care visit could have been cles of respiration to expand the chest wall. Q.C.’s hyperinflated
prevented. chest and her use of suprasternal, supraclavicular, and intercostal
P1: Trim: 8.375in × 10.875in Top: 0.373in Gutter: 0.664in
LWBK915-23 LWW-KodaKimble-educational October 27, 2011 8:22

574
Persistent Asthma: Daily Medication
Intermittent
Asthma Consult with asthma specialist if Step 3 care or higher is required.
Consider consultation at Step 2.

Step 6
Step 5 Step up if
needed
(first, check
Step 4 Preferred: adherence,
Preferred: High-dose inhaler
Step 3 High-dose ICS + either technique, and
Preferred: ICS + either LABA or environmental
Medium-dose LABA or Montelukast control)
Step 2 ICS + either
Preferred: Montelukast
LABA or Oral systemic
Medium-dose
Step 1 Preferred: Montelukast corticosteroids
ICS Assess
Section 3

Low-dose ICS control


Preferred:
SABA PRN Alternative:
Cromolyn or Step down if
Montelukast possible
(and asthma is
Pulmonary Disorders

well controlled
Patient Education and Environmental Control at Each Step at least
3 months)

Quick-Relief Medication for All Patients


• SABA as needed for symptoms. Intensity of treatment depends on severity of symptoms.
• With viral respiratory infection SABA every 4–6 hours up to 24 hours (longer with physician consult).
Consider short course of oral systemic corticosteroids if exacerbation is severe or patient has history
of previous severe exacerbations.
• Caution: Frequent use of SABA may indicate the need to step up treatment. See text for
recommendations on initiating daily long-term-control therapy.

Key: Alphabetical order is used when more than one treatment option is listed within either preferred or
alternative therapy. ICS, inhaled corticosteroid; LABA, inhaled long-acting β2-agonist; SABA, inhaled
short-acting β2-agonist.

Notes:
• The stepwise approach is meant to assist, not replace, the clinical decision-making required to meet individual patient needs.
• If alternative treatment is used and response is inadequate, discontinue it and use the preferred treatment before stepping up.
• If clear benefit is not observed within 4–6 weeks and patient/family medication technique and adherence are satisfactory, consider
adjusting therapy or alternative diagnosis.
• Studies on children 0–4 years of age are limited. Step 2 preferred therapy is based on Evidence A. All other recommendations are
based on expert opinion and extrapolation from studies in older children.

FIGURE 23-7 Stepwise approach for managing asthma in children 0 to 4 years of age. Reprinted from National Institutes of
Health. Expert Panel Report 3: Guidelines for the Diagnosis and Management of Asthma. Bethesda, MD: National Heart, Lung,
and Blood Institute; 2007. NIH publication 07-4051.

muscles to assist in breathing also are compatible with obstruc- monary artery for gas exchange. When ventilation is decreased
tive airway diseases. to an area of the lung, the alveoli in that area become hypoxic,
Occlusion of the small airways, air trapping, and resorption and the pulmonary artery to that region constricts as a normal
of air distal to the obstruction can lead to atelectasis (incomplete physiologic response. As a result, blood flow is shunted to the
expansion or collapse of pulmonary alveoli or of a segment of well-ventilated portions of the lung because of the need to pre-
a lobe of the lung). Localized areas of atelectasis often are dif- serve adequate oxygenation of the blood. The pulmonary arter-
ficult to distinguish from infiltrates on a chest radiograph, and ies, however, are not constricted completely, and when a small
atelectasis can be mistaken for pneumonia. amount of blood flows to the poorly ventilated alveoli, mismatch-
Q.C.’s history of multiple bouts of “bronchitis” is significant ing is the result. Conditions of diffuse bronchial obstruction (i.e.,
and typical of many young asthmatic patients. In any patient with acute asthma) increase the amount of mismatching. In addition,
recurring episodes of bronchial symptoms (i.e., bronchitis, pneu- some mediators of acute bronchospasm (e.g., histamine) further
monia), the possible diagnosis of asthma should be investigated. worsen mismatching by constricting bronchial smooth muscle
The increased pulse, RR, and anxiety experienced by Q.C. can while concurrently relaxing vascular smooth muscle.
be attributed both to hypoxemia and the feeling of suffocation. Q.C. also demonstrated a significant pulsus paradoxus. Pulsus
The hypoxemia in acute asthma is caused principally by an imbal- paradoxus is defined as a drop in systolic BP of more than 10 mm
ance between alveolar ventilation and pulmonary capillary blood Hg with inspiration. In general, pulsus paradoxus correlates with
flow, also known as ventilation–perfusion (V̇/Q̇) mismatching.20 the severity of bronchial obstruction; however, it is not always
Each alveolus of the lung is supplied with capillaries from the pul- present.20
P1: Trim: 8.375in × 10.875in Top: 0.373in Gutter: 0.664in
LWBK915-23 LWW-KodaKimble-educational October 27, 2011 8:22

575
Persistent Asthma: Daily Medication
Intermittent
Asthma Consult with asthma specialist if Step 4 care or higher is required.
Consider consultation at Step 2.

Step 6
Step up if
Step 5 needed
(first, check
Preferred: adherence,
Step 4 Preferred: High-dose ICS inhaler
High-dose + LABA + oral technique,
Step 3 Preferred: ICS + LABA systemic environmental
Medium-dose corticosteroid control, and
Step 2 Preferred:
ICS + LABA Alternative: comorbid
EITHER: Alternative: conditions)
Preferred: Low-dose ICS Alternative: High-dose
Step 1 Low-dose ICS + either LABA, ICS + either High-dose ICS

Chapter 23
LTRA, or Medium-dose LTRA or + either LTRA Assess
Preferred: Alternative: Theophylline Theophylline or Theophylline control
ICS + either
SABA PRN Cromolyn, LTRA or + oral systemic
LTRA, OR: Theophylline corticosteroid
Nedocromil, Step down if
or Theophylline Medium-dose
possible
ICS
(and asthima is

Asthma
well controlled
Each step: Patient education, environmental control, and management of comorbidities. at least
3 months)
Steps 2–4: Consider subcutaneous allergen immunotherapy for patients who have allergic asthma (see notes).

Quick-Relief Medication for All Patients

• SABA as needed for symptoms. Intensity of treatment depends on severity of symptoms: up to 3


treatments at 20-minute intervals as needed. Short course of oral systemic corticosteroids may be
needed.
• Caution: Increasing use of SABA or use >2 days a week for symptom relief (not prevention of EIB)
generally indicates inadequate control and the need to step up treatment.

Key: Alphabetical order is used when more than one treatment option is listed within either preferred or
alternative therapy. ICS, inhaled corticosteroid; LABA, inhaled long-acting β 2-agonist; LTRA, leukotriene
receptor antagonist; SABA, inhaled short-acting β2-agonist.
Notes:
• The stepwise approach is meant to assist, not replace, the clinical decision-making required to meet individual patient needs.
• If alternative treatment is used and response is inadequate, discontinue it and use the preferred treatment before stepping up.
• Theophylline is a less desirable alternative because of the need to monitor serum concentration levels.

FIGURE 23-8 Stepwise approach for managing asthma in children 5 to 11 years of age. Reprinted from National Institutes of
Health. Expert Panel Report 3: Guidelines for the Diagnosis and Management of Asthma. Bethesda, MD: National Heart, Lung,
and Blood Institute; 2007. NIH publication 07-4051.

EXTENT OF OBSTRUCTION airway obstruction and the response to bronchodilator therapy.


Unfortunately, infants and many young children do not have
CASE 23-1, QUESTION 2: What additional tests would be the cognitive or motor skills necessary to perform pulmonary
helpful in assessing the extent of pulmonary obstruction in function tests. EPR-3 points out that in one study, only 65% of
Q.C.? children 5 to 16 years of age could complete either FEV1 or
PEF during an acute exacerbation. Because of Q.C.’s initial anx-
Chest radiographs are not recommended routinely but should iety, the PEF should be measured after bronchodilator therapy
be obtained in patients who are suspected of having a complica- has been initiated when she may be calmer. One disadvantage
tion (e.g., pneumonia).1 Hyperinflated lungs and areas of atelec- of pulmonary function tests in acute asthma is that the forced
tasis can be seen on a chest x-ray film; however, chest x-ray studies expiratory maneuver commonly triggers coughing. ABG mea-
usually are negative and of little value in evaluating acute asthma surements are the gold standard for assessing very severe airway
attacks. The finding of a local decrease in breath sounds in Q.C.’s obstruction.21 EPR-31 suggests ABGs for evaluation of Paco2
left lung may justify the need for a chest x-ray study, particularly in patients who have suspected hypoventilation, severe distress,
if a significant differential in air movement persists after initial or FEV1 or PEF less than 25% of predicted after initial treat-
therapy. A local decrease in breath sounds may indicate pneu- ment. However, in less severe exacerbations, ABG measurements
monia, aspiration of a foreign object, pneumothorax, or merely are unnecessary if other objective measures of airway obstruc-
thickened mucous plugging of a large bronchus. tion (e.g., pulmonary function tests) have been monitored.1 In
Pulmonary function testing (e.g., FEV1 , PEF) provides objec- acute asthma, ABG determinations usually indicate hypoxemia
tive measurement of the degree of airway obstruction. Peak flow because of mismatching and hypocapnia with respiratory alka-
meters are helpful in the ED for assessing both the severity of losis because of hyperventilation.21 The degree of hypoxemia
P1: Trim: 8.375in × 10.875in Top: 0.373in Gutter: 0.664in
LWBK915-23 LWW-KodaKimble-educational October 27, 2011 8:22

576
Persistent Asthma: Daily Medication
Intermittent
Asthma Consult with asthma specialist if Step 4 care or higher is required.
Consider consultation at Step 3.

Step 6
Step up if
Step 5 needed
(first, check
Preferred: adherence,
Step 4 Preferred: High-dose ICS environmental
High-dose + LABA + oral control, and
Step 3 Preferred: ICS + LABA corticosteroid comorbid
Medium-dose conditions)
Step 2 Preferred: ICS + LABA AND AND
Preferred: Low-dose ICS
+ LABA Alternative: Consider Consider
Step 1 Low-dose ICS omalizumab omalizumab Assess
OR
Medium-dose for patients for patients control
Section 3

Preferred: Alternative: Medium-dose who have who have


ICS + either
SABA PRN Cromolyn, ICS LTRA, allergies allergies
LTRA, Alternative: theophylline,
nedocromil, or zileuton Step down if
or theophylline Low-dose ICS possible
+ either LTRA, (and asthma is
theophylline, well controlled
Pulmonary Disorders

or zileuton at least
3 months)

Each step: Patient education, environmental control, and management of comorbidities.

Steps 2–4: Consider subcutaneous allergen immunotherapy for patients who have allergic asthma (see notes).

Quick-Relief Medication for All Patients

• SABA as needed for symptoms. Intensity of treatment depends on severity of symptoms: up to 3


treatments at 20-minute intervals as needed. Short course of oral systemic corticosteroids may be
needed.
• Use of SABA >2 days a week for symptom relief (not prevention of EIB) generally indicates
inadequate control and the need to step up treatment.

Key: Alphabetical order is used when more than one treatment option is listed within either preferred or alternative
therapy. EIB, exercise-induced bronchospasm; ICS, inhaled corticosteroid; LABA, long-acting inhaled β2-agonist;
LTRA, leukotriene receptor antagonist; SABA, inhaled short-acting β 2-agonist.

Notes:
• The stepwise approach is meant to assist, not replace, the clinical decision-making required to meet individual patient needs.
• If alternative treatment is used and response is inadequate, discontinue it and use the preferred treatment before stepping up.
• Zileuton is a less desirable alternative because of limited studies as adjunctive therapy and the need to monitor liver function.
Theophylline requires monitoring of serum concentration levels.
• In step 6, before oral systemic corticosteroids are introduced, a trial of high-dose ICS + LABA + either LTRA,
theophylline, or zileuton may be considered, although this approach has not been studied in clinical trials.

FIGURE 23-9 Stepwise approach for managing asthma in youth 12 years of age or older and adults. Reprinted from
National Institutes of Health. Expert Panel Report 3: Guidelines for the Diagnosis and Management of Asthma. Bethesda,
MD: National Heart, Lung, and Blood Institute; 2007. NIH publication 07-4051.

correlates with the severity of obstruction. Severe hypoxemia NEED FOR HOSPITALIZATION
(Pao2 <50 mm Hg) that is associated with an FEV1 less than
15% of predicted represents very severe airway obstruction.20,21 CASE 23-1, QUESTION 3: Q.C. may require hospitalization.
Likewise, when the FEV1 is less than 25% of the predicted value, Which clinical test is predictive of the need for admission
carbon dioxide increasingly is retained and the Paco2 begins to or whether Q.C. will relapse if sent home from the ED? Are
rise into the usual normal range.1 Because of mismatching and Q.C.’s signs and symptoms predictive of whether she will
the ease of correction of hypoxemia with oxygen therapy, the relapse and return to the ED if not hospitalized?
Paco2 is the more sensitive indicator of ventilation abnormalities
in acute asthma with prolonged or chronic airway obstruction; The most useful predictive tool is the FEV1 or PEF response
carbon dioxide retention (hypercapnia) and respiratory acidosis to initial treatment. Patients who do not improve to at least 40%
are prominent. ABG measurements are indicated in patients who of predicted FEV1 or PEF after initial intensive therapy are more
fail to respond adequately to initial therapy or in patients requir- likely to require hospitalization.1 Although Q.C. is not able to
ing hospitalization; they are not indicated at this time for Q.C. A perform spirometry, she is able to execute the PEF maneuver,
repeat pulse oximetry at 1 hour after treatment initiation is war- and the plan is to check her PEF after 1 hour of therapy. Signs
ranted in Q.C. to ensure adequate arterial oxygen saturation. and symptom scores alone are not adequate to predict outcome
P1: Trim: 8.375in × 10.875in Top: 0.373in Gutter: 0.664in
LWBK915-23 LWW-KodaKimble-educational October 27, 2011 8:22

of ED treatment of asthma, but scores along with pulse oximetry sion or solution mixed with propellant. The valve controls the 577
and PEF or FEV1 are helpful predictors.1 delivery of drug and allows the precise release of a premeasured
amount of the product. A second aerosol device, the air jet neb-
ulizer, mechanically produces a mist of drug. The drug is placed
Short-Acting Inhaled β2 -Adrenergic in a small volume of solute (typically 3 mL of saline) and then
Agonist Therapy placed in a small reservoir (nebulizer) connected to an air source
such as a small compressor pump, oxygen tank, or wall air hose.
SHORT-ACTING INHALED β2 -AGONISTS COMPARED Air travels from the relatively large-diameter tubing of the air
WITH OTHER BRONCHODILATORS source into a pinhole-sized opening in the nebulizer. This creates
a negative pressure at the site of the air entry and causes the drug
CASE 23-1, QUESTION 4: Why was a SABA selected as solution in the bottom of the nebulizer reservoir to be drawn
the bronchodilator of first choice in preference to other up through a small capillary tube where it then encounters the
bronchodilators such as aminophylline or ipratropium for rapid airflow. The drug solution is forced against a small baffle
Q.C.? that causes mechanical formation of a mist. An ultrasonic nebu-
lizer is a type of nebulizer that uses sound waves to generate the
Because of their potency and rapidity of action, inhaled

Chapter 23
aerosol.
β2 -agonists are considered the first choice for the treatment of Studies that compared nebulization with pressurized
acute asthma.1–5 The bronchodilatory properties of SABAs are metered-dose aerosols in stable chronic asthma patients have
particularly effective in reversing early-phase asthma responses. shown no advantage among these methods of administration
Aminophylline (a theophylline salt) is not as efficacious and has when equivalent doses are administered.25,26 Each method deliv-
more risks for serious adverse effects than inhaled albuterol.1–5 ers approximately 10% of the beginning dose to the patient’s
Similarly, the bronchodilation from the anticholinergic drug ipra- airways.27 Trials comparing metered-dose aerosols of inhaled

Asthma
tropium is of smaller magnitude than with inhaled SABAs.1–5 SABAs with the nebulization of those same drugs in acute asthma
However, two double-blind pediatric trials found that the sickest also have shown no significant advantage for the nebulization
children had a reduced rate of hospitalization if given ipratropium method of administration when the metered-dose aerosolized
with albuterol in the ED.22,23 In one trial,22 children with base- administration was carefully supervised by experienced per-
line FEV1 less than 30% of predicted value had a reduced rate sonnel and a spacer device was used.26,28,29 However, in some
of admission with ipratropium, and in the other trial,23 children younger acutely ill children, it is difficult (even with supervision)
with baseline PEF less than 50% had a reduced rate of hospi- to administer an effective SABA with a metered-dose canister.
tal admission. Consequently, although early addition of inhaled Because many patients and clinicians perceive that nebulizers pro-
ipratropium in adequate doses to SABAs will improve pulmonary vide more intensive therapy, it often is important psychologically
function tests and reduce the rate of hospitalization in severely to give at least the first dose of a SABA via a nebulizer. Thereafter,
ill patients, Q.C.’s physician chose to use only inhaled SABAs it is more cost-effective to use the therapeutically equivalent MDI
initially because Q.C. was not severely ill. plus spacer.30
The dose ratio for SABAs delivered by MDI plus spacer versus
PREFERRED ROUTES OF ADMINISTRATION
nebulizer has varied in the literature. For children with mild
acute asthma, 2 puffs of albuterol MDI attached to a spacer
were not different from 6 to 10 puffs of albuterol or via neb-
CASE 23-1, QUESTION 5: What is the preferred route of
ulizer 0.15 mg/kg.31 In one double-blind trial in children with
administration for short-acting bronchodilators?
a severe exacerbation, investigators used a dose ratio of 1 : 5
It is well documented that SABAs administered by the inhaled (i.e., albuterol MDI-spacer 1 mg [10 puffs]: nebulized albuterol
route provide as great or greater bronchodilation with fewer sys- 5 mg).32 Nebulization of albuterol with compressed air or, prefer-
temic side effects than either the parenteral or oral routes.1–6 ably, oxygen was the preferred method of administration for Q.C.
In situations of acute bronchospasm, concerns about adequate initially.
penetration of aerosols into the bronchial tree led many clini-
DOSING
cians to believe that the parenteral route of administration would
be more effective than the inhaled route of administration. In
CASE 23-1, QUESTION 7: Starting 20 minutes after the first
clinical trials, however, inhaled SABAs were as effective as the
albuterol dose, two more doses of 2.5 mg of albuterol were
standard treatment of subcutaneous epinephrine for ED treat-
administered by nebulizer every 20 minutes during the next
ment of acute asthma in adults and children.1–6,24 Therefore,
40 minutes. After three treatments, Q.C.’s breath sounds
aerosolized SABAs now are considered the agents of choice for
became increasingly clear. She was no longer in distress and
ED or hospital management of asthma.1 β 2 -Agonists should not
could speak in complete sentences. Her PEF was now 70%
be administered orally to treat acute episodes of severe asthma
of predicted, her SaO2 was 97% on room air, and discharge
because of the slow onset of action, lower efficacy, and erratic
to home was planned. Were the dose and dosing interval of
absorption.1
albuterol appropriate for Q.C.?

CASE 23-1, QUESTION 6: Q.C. received albuterol by nebu- Schuh et al.33 demonstrated that a higher-dose albuterol
lization. Would metered-dose aerosol administration of the regimen (0.15 mg/kg vs. 0.05 mg/kg every 20 minutes) pro-
SABA have been preferred? Is the dose given by nebu- duced significantly greater improvement with no greater inci-
lization the same as that given by a metered-dose inhaler dence of adverse effects. Schuh et al.34 subsequently reported
(MDI)? greater efficacy of albuterol in a dose of 0.3 mg/kg (up to 10
mg) hourly compared with a dose of 0.15 mg/kg (up to 5 mg)
Aerosols are mixtures of particles (e.g., a drug–lipid mixture) hourly in children. The larger dose was tolerated as well as
suspended in a gas. An MDI consists of an aerosol canister and the 0.15 mg/kg dose. Therefore, Q.C.’s albuterol regimen of
an actuation device (valve). The drug in the canister is a suspen- 2.5 mg (0.13 mg/kg) nebulized every 20 minutes for 40 minutes
P1: Trim: 8.375in × 10.875in Top: 0.373in Gutter: 0.664in
LWBK915-23 LWW-KodaKimble-educational October 27, 2011 8:22

578
Initial Assessment
Brief history, physical examination (auscultation, use of accessory muscles, heart rate,
respiratory rate), PEF or FEV1, oxygen saturation, and other tests as indicated.

FEV1 or PEF 40% (Mild-to-Moderate) FEV1 or PEF <40% (Severe) Impending or Actual
Oxygen to achieve SaO 2 90% Oxygen to achieve SaO 2 90% Respiratory Arrest
Inhaled SABA by nebulizer or MDI with High-dose inhaled SABA plus Intubation and mechanical
valved holding chamber, up to 3 doses in ipratropium by nebulizer or MDI ventilation with 100% oxygen
first hour plus valved holding chamber, Nebulized SABA and ipratropium
Oral systemic corticosteroids if no immediate every 20 minutes or continuously Intravenous corticosteroids
response or if patient recently took oral for 1 hour Consider adjunct therapies
systemic corticosteroids Oral systemic corticosteroids

Repeat Assessment Admit to Hospital Intensive Care


Symptoms, physical examination, PEF, O2 saturation, other tests (see box below)
Section 3

as needed

Moderate Exacerbation Severe Exacerbation


FEV1 or PEF 40%–69% FEV1 or PEF <40% predicted/personal best
predicted/personal best Physical examination: severe symptoms at rest, accessory muscle
Pulmonary Disorders

Physical examination: moderate use, chest retraction


symptoms History: high-risk patient
Inhaled SABA every 60 minutes No improvement after initial treatment
Oral systemic corticosteroid Oxygen
Continue treatment 1–3 hours, Nebulized SABA + ipratropium, hourly or continuous
provided there is improvement; Oral systemic corticosteroids
make admit decision in <4 hours Consider adjunct therapies

Good Response Incomplete Response Poor Response


FEV1 or PEF 70% FEV1 or PEF 40%–69% FEV1 or PEF <40%
Response sustained 60 minutes Mild-to-moderate symptoms PC O 2 42 mm Hg
after last treatment Physical examination:
No distress symptoms severe,
Physical exam: normal Individualized decision re: drowsiness, confusion
hospitalization

Discharge Home Admit to Hospital Ward Admit to Hospital Intensive Care


Continue treatment with inhaled SABA. Oxygen Oxygen
Continue course of oral systemic corticosteroid. Inhaled SABA Inhaled SABA hourly or continuously
Consider initiation of an ICS. Systemic (oral or intravenous) Intravenous corticosteroid
Patient education corticosteroid Consider adjunct therapies
– Review medications, including inhaler technique. Consider adjunct therapies Possible intubation and mechanical
– Review/initiate action plan. Monitor vital signs, FEV1 or ventilation
– Recommend close medical follow-up. PEF, SaO 2

Improve Improve

Discharge Home
Continue treatment with inhaled SABAs.
Continue course of oral systemic corticosteroid.
Continue on ICS. For those not on long-term control therapy, consider initiation of an ICS.
Patient education (e.g., review medications, including inhaler technique and, whenever possible,
environmental control measures; review/initiate action plan; recommend close medical follow-up).
Before discharge, schedule follow-up appointment with primary-care provider or asthma
specialist in 1–4 weeks.

FIGURE 23-10 Management of asthma exacerbations: emergency department and hospital-based care. FEV1 , forced
expiratory volume in 1 second; ICS, inhaled corticosteroid; MDI, metered-dose inhaler; PCO2 , partial pressure of carbon dioxide;
PEF, peak expiratory flow; SABA, short-acting β 2 -agonist; SaO2 , arterial oxygen saturation. Adapted from National Institutes of
Health. Expert Panel Report 3: Guidelines for the Diagnosis and Management of Asthma. Bethesda, MD: National Heart, Lung,
and Blood Institute; 2007. NIH publication 07-4051.

subsequent to her first dose of aerosolized albuterol could have COMPARISON OF SHORT-ACTING
been even more aggressive but was appropriate. Figure 23-10 INHALED β2 -AGONISTS
and Table 23-5 provide treatment guidelines for the manage-
ment of acute asthma exacerbations.1 In addition to EPR-3, a CASE 23-1, QUESTION 8: Would another SABA have been
review of the emergency treatment of asthma has recently been more effective in the initial therapy of Q.C.?
published.35
P1: Trim: 8.375in × 10.875in Top: 0.373in Gutter: 0.664in
LWBK915-23 LWW-KodaKimble-educational October 27, 2011 8:22

579
TA B L E 2 3 - 5
Dosages of Drugs for Asthma Exacerbations

Dosages
Medication Child Dosea Adult Dose Comments

Inhaled Short-Acting β2 -Agonists

Albuterol
Nebulizer solution (0.63 mg/3 mL, 0.15 mg/kg (minimum dose 2.5 mg) 2.5–5 mg every 20 minutes for Only selective β 2 -agonists are
1.25 mg/3 mL, 2.5 mg/3 mL, every 20 minutes for 3 doses, then 3 doses, then 2.5–10 mg recommended. For optimal
5.0 mg/mL) 0.15–0.3 mg/kg up to 10 mg every every 1–4 hours as needed delivery, dilute aerosols to
1–4 hours as needed or 0.5 mg/kg/h or 10–15 mg/h minimum of 3 mL at gas flow of
by continuous nebulization. continuously. 6–8 L/min. Use large-volume
nebulizers for continuous
administration. May mix with

Chapter 23
ipratropium nebulizer solution.
MDI (90 mcg/puff ) 4–8 puffs every 20 minutes for 3 doses, 4–8 puffs every 20 minutes up In mild to moderate exacerbations,
then every 1–4 hours inhalation to 4 hours, then every MDI plus VHC is as effective as
maneuver as needed. Use VHC; 1–4 hours as needed. nebulized therapy with appropriate
add mask in children <4 years. administration technique and
coaching by trained personnel.
Levalbuterol

Asthma
(R-albuterol)
Nebulizer solution (0.63 mg/3 mL, 0.075 mg/kg (minimum dose 1.25 mg) 1.25–2.5 mg every 20 minutes Levalbuterol administered in one-half
1.25 mg/0.5 mL, 1.25 mg/3 mL) every 20 minutes for 3 doses, then for 3 doses, then 1.25–5 mg the mg dose of albuterol provides
0.075–0.15 mg/kg up to 5 mg every every 1–4 hours as needed. comparable efficacy and safety. Has
1–4 hours as needed. not been evaluated by continuous
nebulization.
MDI (45 mcg/puff ) See albuterol MDI dose. See albuterol MDI dose.
Pirbuterol
MDI (200 mcg/puff ) See albuterol MDI dose; thought to be See albuterol MDI dose. Has not been studied in severe
half as potent as albuterol on a asthma exacerbations.
milligram basis.
Systemic (Injected) β2 -Agonists

Epinephrine 1:1,000 (1 mg/mL) 0.01 mg/kg up to 0.3–0.5 mg every 0.3–0.5 mg every 20 minutes No proven advantage of systemic
20 minutes for 3 doses for 3 doses subcutaneously. therapy versus aerosol.
subcutaneously.
Terbutaline (1 mg/mL) 0.01 mg/kg every 20 minutes for 0.25 mg every 20 minutes for No proven advantage of systemic
3 doses, then every 2–6 hours as 3 doses subcutaneously. therapy versus aerosol.
needed subcutaneously.
Anticholinergics

Ipratropium bromide
Nebulizer solution (0.25 mg/mL) 0.25–0.5 mg every 20 minutes for 0.5 mg every 20 minutes for May mix in same nebulizer with
3 doses, then as needed. 3 doses, then as needed. albuterol. Should not be used as
first-line therapy; should be added
to SABA therapy for severe
exacerbations. The addition of
ipratropium has not been shown to
provide further benefit once the
patient is hospitalized.
MDI (18 mcg/puff ) 4–8 puffs every 20 minutes as needed 8 puffs every 20 minutes as Should use with VHC and face mask
up to 3 hours. needed up to 3 hours. for children <4 years.
Studies have examined ipratropium
bromide MDI for up to 3 hours.

Ipratropium with albuterol


Nebulizer solution (Each 3 mL vial 1.5 mL every 20 minutes for 3 doses, 3 ml every 20 minutes for May be used for up to 3 hours in the
contains 0.5 mg ipratropium then as needed. 3 doses, then as needed. initial management of severe
bromide and 2.5 mg albuterol.) exacerbations. The addition of
ipratropium to albuterol has not
been shown to provide further
benefit once the patient is
hospitalized.
MDI (Each puff contains 18 mcg 4–8 puffs every 20 minutes as needed 8 puffs every 20 minutes as Should use with VHC and face mask
ipratropium bromide and up to 3 hours. needed up to 3 hours. for children <4 years.
90 mcg albuterol.)
(continued)
P1: Trim: 8.375in × 10.875in Top: 0.373in Gutter: 0.664in
LWBK915-23 LWW-KodaKimble-educational October 27, 2011 8:22

580
TA B L E 2 3 - 5
Dosages of Drugs for Asthma Exacerbations (Continued )

Dosages
Medication Child Dosea Adult Dose Comments

Systemic Corticosteroids

(Applies to all three corticosteroids)


Prednisone 1 mg/kg in 2 divided doses 40–80 mg/day in 1 or 2 For outpatient “burst,” use 40–60 mg in single
Methyl prednisolone (maximum = 60 mg/day) divided doses until PEF or 2 divided doses for total of 5–10 days in
Prednisolone until PEF is 70% of reaches 70% of predicted adults (children: 1–2 mg/kg/day maximum
predicted or personal best. or personal best. 60 mg/day for 3–10 days).
a
Children, 12 years of age or younger.
MDI, metered-dose inhaler; PEF, peak expiratory flow; SABA, short-acting β2 -agonist; VHC, valved holding chamber.
Reprinted from National Institutes of Health. Expert Panel Report 3: Guidelines for the Diagnosis and Management of Asthma. Bethesda, MD: National Heart, Lung, and Blood
Section 3

Institute; 2007. NIH publication 07-4051.

SABAs (e.g., albuterol) are preferred over nonspecific ago- ADVERSE EFFECTS
nists (e.g., isoproterenol). Long-acting β 2 -agonists (e.g., salme-
CASE 23-2
Pulmonary Disorders

terol) are not indicated for treatment of asthma in the ED. Lev-
albuterol (R-albuterol) became available in the late 1990s, but QUESTION 1: H.T., a 45-year-old, 91-kg man with a long his-
further study is needed to determine whether this single-isomer, tory of severe persistent asthma, presents to the ED with
higher-potency drug offers any clinically significant advantages severe dyspnea and wheezing. He is able to say only two
(i.e., improved outcomes) versus racemic albuterol to justify its or three words without taking a breath. He has been tak-
higher cost.1 ing four inhalations of beclomethasone hydrofluoroalkane
(HFA; 80 mcg/puff) twice daily (BID) and two inhalations
of albuterol MDI four times a day (QID) as needed (PRN)
on a chronic basis. H.T. ran out of beclomethasone a week
SYSTEMIC CORTICOSTEROIDS IN ago; since then, he has been using his albuterol MDI with
THE EMERGENCY DEPARTMENT increasing frequency up to every 3 hours on the day before
FOR CHILDREN admission. He is a lifelong nonsmoker. His FEV1 was 25%
of the predicted value for his age and height, and his SaO2
CASE 23-1, QUESTION 9: Should Q.C. receive cortico- was 82%. Vital signs are as follows:
steroid therapy as part of her ED management?
Heart rate, 130 beats/minute
RR, 30/minute
Yes. Because asthma is primarily an inflammatory airway dis- Pulsus paradoxus, 18 mm Hg
ease, the degree of inflammation associated with Q.C.’s current BP, 130/90 mm Hg
exacerbation should be considered. Per EPR-31 (Fig. 23-10), if
ABGs on room air were as follows:
there is not an immediate response to inhaled β 2 -agonist ther-
apy, oral systemic corticosteroids should be administered (refer pH, 7.40
to further discussion of this subject in Case 23-2, Questions 4 and PaO2 , 55 mm Hg
5). Furthermore, if Q.C. had a peak flow meter at home, ear- PaCO2 , 40 mm Hg
lier objective detection of the development of this exacerbation Serum electrolyte concentrations were as follows:
might have prevented an ED visit. When the PEF is in the red Sodium (Na), 140 mEq/L
zone (<50% of personal best) and poorly responsive to SABAs, Potassium (K), 3.9 mEq/L
early intervention with oral corticosteroids is associated with a Chloride (Cl), 105 mEq/L
reduction in ED visits1 (Fig. 23-11; refer to the Outcomes sec- Because of the severity of the obstruction, H.T. was moni-
tion at the end of the chapter). Q.C. and her parents should also tored with an electrocardiogram that showed sinus tachy-
understand that if respiratory distress is severe and nonrespon- cardia with occasional premature ventricular contractions.
sive to treatment, they should proceed to an ED or call 9-1-1. Albuterol 5.0 mg and ipratropium 0.5 mg were adminis-
Finally, before going home from the ED, Q.C. and her parents tered by nebulization with minimal improvement. H.T. then
should receive some basic education regarding asthma and its was started on O2 at 4 L /minute by nasal cannula, fol-
acute and long-term management. It is important to follow up lowed in 20 minutes by a second dose of albuterol 5.0
with more detailed education during future clinic visits. Accord- mg plus ipratropium 0.5 mg via nebulizer. Subsequently,
ing to EPR-3,1 Q.C. should receive a short course of systemic H.T.’s heart rate increased to 140 beats/minute, and he
corticosteroids as part of her discharge plan, thereby reducing complained of palpitations and shakiness. His PEF was
her risk of re-exacerbation. Typically, oral prednisolone solu- now 25% of personal best. Laboratory values were as
tion at a dose of 1 to 2 mg/kg in daily or divided doses twice follows:
daily is given for approximately 5 to 7 days. Although this reg- pH, 7.39
imen is very effective, to improve compliance several studies PaO2 , 60 mm Hg
have examined shorter (1- to 2-day) courses of oral or intra- PaCO2 , 42 mm Hg
muscular dexamethasone and have found similar results when Na, 138 mEq/L
compared with usual regimens of oral prednisone or predni- K, 3.5 mEq/L
solone.36–38
P1: Trim: 8.375in × 10.875in Top: 0.373in Gutter: 0.664in
LWBK915-23 LWW-KodaKimble-educational October 27, 2011 8:22

581
Assess Severity
Patients at high risk for a fatal attack require immediate medical attention after initial treatment.
Symptoms and signs suggestive of a more serious exacerbation such as marked breathlessness,
inability to speak more than short phrases, use of accessory muscles, or drowsiness should result in
initial treatment while immediately consulting with a clinician.
Less severe signs and symptoms can be treated initially with assessment of response to therapy and
further steps as listed below.
If available, measure PEF—values of 50%–79% predicted or personal best indicate the need for quick-relief
medication. Depending on the response to treatment, contact with a clinician may also be indicated.
Values < 50% indicate the need for immediate medical care.

Initial Treatment
Inhaled SABA: up to two treatments 20 minutes apart of 2–6 puffs by

Chapter 23
metered-dose inhaler (MDI) or nebulizer treatments.
Note: Medication delivery is highly variable. Children and individuals
who have exacerbations of lesser severity may need fewer puffs than
suggested above.

Asthma
Good Response Incomplete Response Poor Response
No wheezing or dyspnea Persistent wheezing and Marked wheezing and dyspnea.
(assess tachypnea in young dyspnea (tachypnea). PEF <50% predicted or personal
children). PEF 50%–79% predicted or best.
PEF 80% predicted or personal best. Add oral systemic
personal best. Add oral systemic corticosteroid.
Contact clinician for follow-up corticosteroid. Repeat inhaled SABA
instructions and further Continue inhaled SABA. immediately.
management. Contact clinician urgently If distress is severe and
May continue inhaled SABA (this day) for further nonresponsive to initial
every 3–4 hours for 24–48 instruction. treatment:
hours. – Call your doctor AND
Consider short course of oral – PROCEED TO ED;
systemic corticosteroids. – Consider calling 9-1-1
(ambulance transport).

To ED.

FIGURE 23-11 Management of asthma exacerbations: home treatment. ED, emergency department; MDI,
metered-dose inhaler; PEF, peak expiratory flow; SABA, short-acting β2 -agonist. Adapted from National Institutes of Health.
Expert Panel Report 3: Guidelines for the Diagnosis and Management of Asthma. Bethesda, MD: National Heart, Lung, and
Blood Institute; 2007. NIH publication 07-4051.

effects may be more noticeable with high doses of inhaled


What adverse effects experienced by H.T. are consistent
albuterol. A β 2 -adrenergic–mediated increase in glucose and
with systemic β2 -agonist administration?
insulin secretion also can contribute to the intracellular shift
H.T. experienced palpitations, which may have been caused by of potassium.39 The shakiness (tremors) experienced by H.T.
the widening of his pulse pressure from vasodilation.3 Albuterol probably can be attributed to β 2 -receptor stimulation of skeletal
and all other β-agonists are cardiac stimulants that may cause muscle. Again, this effect is most prominent with oral or par-
tachycardia and, very rarely, arrhythmias. Because they are rel- enteral administration, but some patients are very sensitive to
atively β 2 -specific, the cardiac effects are more prominent with even small doses of inhaled SABAs.
systemic administration (as opposed to inhalation) and at higher
dosages. However, other causes of cardiac effects must also be β -ADRENERGIC AGONIST SUBSENSITIVITY
considered, such as hypoxemia, which is also a potent stimulus
for cardiac arrhythmias. Therefore, H.T.’s tachycardia may have CASE 23-2, QUESTION 2: Why did H.T. fail to respond to
been caused by the β 2 -agonist, by the worsening of his airway the initial therapy? Could tolerance to the β2 -agonists have
obstruction (as reflected in the increase in Paco2 ), or by both of contributed?
these variables.
The decrease in the serum potassium concentration from Although tolerance to systemic effects of β 2 -agonists (e.g.,
3.9 mEq/L to 3.5 mEq/L could be attributed to β 2 -adrenergic acti- tremor, sleep disturbances) is documented, tolerance to the air-
vation of the Na+ -K+ pump and subsequent transport of potas- way response does not occur to a clinically significant extent.41
sium intracellularly.39,40 However, at usual doses, aerosolized Even with long-term use, the intensity of response to β 2 -agonists
albuterol causes relatively little effect on serum potassium. The is retained (i.e., the maximal percent increase in pulmonary
P1: Trim: 8.375in × 10.875in Top: 0.373in Gutter: 0.664in
LWBK915-23 LWW-KodaKimble-educational October 27, 2011 8:22

582 function), but the duration of response with each dose may
CASE 23-2, QUESTION 5: What would be an appropriate
shorten. Such an effect is unlikely with intermittent use but may
dosing regimen of corticosteroids for H.T. in the ED? Would
occur in patients who routinely use large, multiple doses daily.
the dose and route be the same if he were hospitalized?
Possible explanations for this variability include downregulation
of receptors, disease progression, or true drug tolerance. The
Doses of corticosteroids used to treat acute asthma are largely
exact contribution of each is not known. Therefore, H.T.’s fail-
empiric. Studies comparing very high dosages (e.g., intravenous
ure to respond to the initial therapy most likely is attributable
[IV] methylprednisolone 125 mg every 6 hours in an adult) ver-
to the severity of his airway obstruction. H.T.’s history of severe
sus moderate dosages (40 mg every 6 hours) have shown no
chronic asthma, the slow progression of this attack, and the lack
advantage with very high dosages.1,52,53 In addition, oral ther-
of response to his inhaled β 2 -agonist also are largely owing to a
apy is as efficacious as the IV route.1,52,53 Dosing recommenda-
significant inflammatory component to this attack. Thus, bron-
tions for systemic corticosteroids in the management of asthma
chodilators would not be expected to immediately reverse the
exacerbations in the ED or hospital are shown in Table 23-5.
airway obstruction in H.T. It would be difficult to attribute his
Higher corticosteroid dosages may be considered in patients with
lack of initial response to therapy to β 2 -adrenergic subsensitivity.
impending respiratory failure. When patients cannot take oral
In addition, it is not likely that β 2 -adrenergic receptor polymor-
medication, IV methylprednisolone is preferred over hydrocor-
phisms could account for H.T.’s initial lack of response.42
Section 3

tisone in patients with heart disease or fluid retention or when


Although polymorphic variations are documented to be rele-
high dosages of corticosteroids are used; this is because it has less
vant in some stable patients,1 further study is needed to establish
mineralocorticoid activity.
clinical relevance.
For patients who require IV corticosteroid therapy, the dosage
can usually be reduced rapidly to 60 to 80 mg/day for adults
CASE 23-2, QUESTION 3: Repeat measurements of PEF and
(1–2 mg/kg/day for children) as the condition improves (usu-
ABGs indicate continued significant bronchial obstruction.
Pulmonary Disorders

ally after 48–72 hours). On discharge from the hospital, EPR-31


What should be the next step in H.T.’s therapy?
recommends, for example, prednisone 40 to 80 mg/day in one
or two doses for 3 to 10 days. Although some clinicians may
H.T. received initial therapy with a combination of inhaled
prescribe a tapering regimen, no taper is necessary in this situ-
SABAs and ipratropium. He should receive a third dose of
ation. On the other hand, if the patient required long-term oral
albuterol 5.0 mg plus ipratropium 0.5 mg 20 minutes after the
corticosteroid therapy before hospitalization, tapering the dose
second dose. Because of the severity of his exacerbation, he
to the preadmission dosage is prudent. For patients who are dis-
should then receive albuterol 15 mg per hour by continuous
charged from the ED, up to 7 days of prednisone therapy usually is
nebulization with close monitoring of his cardiac status. H.T.’s
sufficient.
PEF also should be monitored.

SHORT-ACTING INHALED β -AGONISTS IN CASE 23-2, QUESTION 6: H.T. was given one dose of 60 mg
COMBINATION WITH CORTICOSTEROIDS of methylprednisolone (Solu-Medrol) IV and three doses
of albuterol 5 mg/ipratropium 0.5 mg by nebulizer every
CASE 23-2, QUESTION 4: Are systemic corticosteroids 20 minutes in the ED). H.T. claimed slight subjective
appropriate for H.T.? When can a response be expected? improvement after this therapy; yet, expiratory wheezes
still were audible, and he still was using his accessory mus-
Corticosteroids have potent anti-inflammatory activity and cles for ventilatory efforts. His PEF improved only to 35% of
are definitely indicated in H.T.1–6 In patients like H.T. with acute predicted, and a repeat ABG measurement showed a PaCO2
asthma, corticosteroids decrease airway inflammation43–46 and of 40 mm Hg. What should be done at this time?
increase the response to β 2 -selective agonists.1,43 Corticosteroids
are not smooth muscle relaxants (i.e., not bronchodilators); H.T. still is significantly obstructed despite intensive therapy
however, they can relieve bronchial obstruction by improving in the ED. As a result, he should be admitted to the intensive care
the responsiveness of β 2 -receptors and by inhibiting numerous unit (ICU), where he can be monitored closely.
phases of the inflammatory response (e.g., cytokine production,
neutrophil and eosinophil chemotaxis and migration, and release
of inflammatory mediators).1–6
Respiratory Failure
The anti-inflammatory activity of corticosteroids is delayed SIGNS AND SYMPTOMS
for about 4 to 6 hours after the dose has been adminis-
tered. Corticosteroid-induced restoration of responsiveness to CASE 23-2, QUESTION 7: What would be the best method
endogenous catecholamines and exogenous β 2 -agonists, how- of assessing the adequacy of therapy in H.T.? What are the
ever, occurs within 1 hour of administration of the cortico- signs of impending respiratory failure?
steroid in severe, chronic, stable asthmatic patients.43 Signifi-
cant improvement in objective measures (e.g., FEV1 ) generally When patients continually must expand their chest wall with
occurs 12 hours after administration.45 Consequently, EPR-31 high lung volumes for a prolonged period, respiratory muscle
advocates early initiation of corticosteroids in cases of acute fatigue may ensue, resulting in a decreased ventilatory effort.
severe asthma. Corticosteroids also hasten the recovery of acute Clinical signs of impending respiratory failure include increased
exacerbations of asthma47–51 and decrease the need for hospital- heart rate, decreased breath sounds, agitation from worsening
ization if given early in the initial management of acute asthma in hypoxia, or lethargy from increased CO2 retention. These clinical
the ED.46 signs and symptoms are relatively nonspecific and are affected by
On the basis of his initial presentation, H.T. should be started many variables. Thus, they should not be used to detect impend-
on systemic corticosteroid therapy immediately in the ED (Fig. ing respiratory failure.
23-10). Preferably, oral corticosteroids would have been started The best way to assess therapy is to monitor ABGs. The Pao2
at home before H.T.’s exacerbation escalated to this degree of component of an ABG determination is not very helpful because
severity (Fig. 23-11). of V̇/Q̇ mismatching and the administration of oxygen. The
P1: Trim: 8.375in × 10.875in Top: 0.373in Gutter: 0.664in
LWBK915-23 LWW-KodaKimble-educational October 27, 2011 8:22

Paco2 is the best indicator of hypoventilation in acute asthma1 ; of theophylline in the hospital.1 H.T.’s clinicians decide not to 583
however, there is no single value for Paco2 that indicates impend- use theophylline. If a clinician decides to use theophylline in this
ing respiratory failure, because different Paco2 values are accept- setting, a current pharmacokinetics text should be consulted to
able under different clinical circumstances. A Paco2 of 55 mm Hg help ensure safe and effective dosing and monitoring.67
1 to 2 hours after intensive bronchodilator therapy or an increase
in Paco2 of 5 to 10 mm Hg/hour during aggressive therapy is RESPONSE TO THERAPY
an ominous sign. The fact that H.T.’s Paco2 is not rising with
therapy is a good sign. CASE 23-2, QUESTION 10: H.T. has continued to improve
slowly during the last 72 hours. The nebulizer treatments
β2 -AGONISTS AND OTHER POTENTIAL THERAPIES with albuterol are now administered every 4 hours, and he
is taking oral prednisone 80 mg/day in two divided doses.
CASE 23-2, QUESTION 8: H.T. initially received three doses PEF measurements taken before and after the last albuterol
of albuterol 5 mg/ipratropium 0.5 mg by nebulizer every treatment were 65% of predicted and 80% of predicted,
20 minutes in the ED. He has also received IV methylpred- respectively. Is H.T.’s long duration of recovery unusual?
nisolone 60 mg. Would the IV administration of a β -agonist
No. In a patient such as H.T., whose condition progressively

Chapter 23
be indicated in H.T. at this time? What are other potential
therapies in H.T.? deteriorated during a long period, a slow reversal should be
expected. The prolonged deterioration reflects an increasing
Although the use of IV β-agonists for asthmatic patients in the inflammatory response in the lung. These patients require pro-
ICU formerly was advocated, current standards of care discour- longed, intensive bronchodilator and anti-inflammatory therapy
age the use of these agents.1 H.T.’s response to inhaled albuterol before maximal improvement is noted in pulmonary function
also suggest that IV β-agonists are inappropriate at this time. tests. Thus, H.T. should continue to receive systemic cortico-

Asthma
Because standard therapies already administered were not suffi- steroids for approximately 10 days after such a severe acute exac-
cient, IV magnesium sulfate may benefit severely ill patients like erbation of asthma.1
H.T.1,54 Recent research suggests that nebulized isotonic magne-
sium sulfate is a valuable adjunctive therapy to inhaled albuterol Adverse Effects of Short-Term
in the treatment of severe asthma exacerbations.55 Further, heliox Corticosteroid Therapy
(a mixture of helium and oxygen) may also add benefit.1,56
CASE 23-2, QUESTION 11: H.T. has been taking cortico-
THEOPHYLLINE steroids for a total of 6 days. Long-term corticosteroid use
is associated with many adverse effects (e.g., adrenal sup-
CASE 23-2, QUESTION 9: Intravenous theophylline is being
pression, osteoporosis, cataracts). What adverse effects are
considered for H.T. Is theophylline likely to be of benefit in
related to short-term corticosteroid use?
impending respiratory failure?
Short courses of daily corticosteroids are usually associated
Studies on the treatment of acute asthma in the ED have with minor side effects.47–51 Facial flushing, appetite stimula-
failed to demonstrate any benefit of adding theophylline to opti- tion, gastrointestinal irritation, headache, and mood changes
mal, inhaled β-agonist therapy,1,57 and the NIH guidelines do ranging from a mere sense of well-being to overt toxic psy-
not recommended this practice.1–6 Further evidence of the lack chosis are the most commonly encountered adverse effects
of value of theophylline in the acute-care setting emerged in of short-term corticosteroid therapy. Acne can be exacerbated
the 1990s. Several double-blind, randomized, placebo-controlled in patients susceptible to this skin problem, and weight gain
studies have demonstrated that theophylline does not add ben- also can occur because of sodium and fluid retention. In addi-
efit to intensive therapy with inhaled β 2 -agonists and systemic tion, hyperglycemia, leukocytosis, and hypokalemia are possi-
corticosteroids in hospitalized adults58 or children59–62 who fail ble. All of these problems are transient and will disappear with
to respond to aggressive ED therapy with SABAs. Note in Fig- time after the corticosteroids are discontinued. These short-
ure 23-10 from EPR-31 that theophylline is not recommended term adverse effects are less common when small cortico-
for routine management of hospitalized patients with asthma. steroid doses are used; however, corticosteroid doses must be
Although one study63 has shown a slight benefit of theophylline adequate to prevent disease exacerbation. The minor risks of
in hospitalized adult asthmatic patients, one of the authors of short-term use are far outweighed by the marked benefits.
that study has since noted that if adequate doses of SABAs and
systemic corticosteroids are used, theophylline is probably not
routinely indicated.64 Although further research is needed to Overuse of Short-Acting Inhaled
establish whether theophylline may add benefit to hospitalized β -Agonists
patients who have impending respiratory failure, routine use
of theophylline in hospitalized asthmatic patients no longer is CASE 23-2, QUESTION 12: H.T.’s history of increased use
justified. of his SABA inhaler during the early stages of this asthma
Two studies suggest a potential benefit of theophylline for attack suggest improper use of this medication. What are
some patients in the ICU who have impending respiratory the risks from overuse of β2 -agonists?
failure.65,66 The authors of these studies point out that if theo-
phylline is used, clinicians must be competent in dosing and The overuse of SABAs as a possible risk factor for asthma death
monitoring serum concentrations. Based on a lack of evidence has been debated for decades, and this debate was revived in the
in several trials in hospitalized asthmatic patients,46–50 more early 1990s.68 Because most deaths attributable to asthma occur
trials are needed to evaluate whether possible benefits outweigh outside the hospital setting before the patient can reach medical
risks of this drug for impending respiratory failure. On the basis assistance, the primary cause of death in asthma most probably
of inconclusive evidence of improved patient response but doc- is an underestimation of the severity of the asthma attack by
umented increased risks, EPR-3 does not recommend the use the patient and delay in seeking medical help. Overuse of quick
P1: Trim: 8.375in × 10.875in Top: 0.373in Gutter: 0.664in
LWBK915-23 LWW-KodaKimble-educational October 27, 2011 8:22

584 reliever medication suggests inadequate asthma control and can of inability to generate sufficient peak inspiratory flow (PIF). As
lead to fatal asthma.1,68 B.C. matures, he may be able to use some dry powder inhalers
The frequency of as-needed doses of albuterol is a good (e.g., Diskus). Because oral β 2 -agonists are not recommended by
marker of the adequacy of inhaled anti-inflammatory therapy EPR-3 and the albuterol syrup was not well tolerated, they should
and environmental control measures. For example, if the patient not be used. If concurrent inhaled albuterol is administered cor-
needs the SABA more than two or three times a day, the clin- rectly, oral albuterol is not needed and would only be expected
ician should reassess environmental control, increase the dose to add adverse effects. B.C.’s mother must be educated about
of inhaled anti-inflammatory therapy, or add other controller asthma, its treatment, and the appropriate use of medications
agents per the NIH guidelines.1–6 (e.g., proper use of inhaler devices; refer to Case 23-13, Ques-
Patients should be instructed verbally and in writing regarding tions 1 and 2).
the proper use of their inhalers during acute attacks and in recog-
nizing when it is necessary to seek medical assistance (Fig. 23-11). CASE 23-3, QUESTION 3: It is anticipated that because of
Patients can continue using their short-acting β-agonist inhalers his young age, B.C. will find it difficult to use an MDI. What
on an as-needed basis until they reach medical care. H.T. should alternative inhalation devices are reasonable?
be considered at high risk because of the severity of his latest
attack and should be given oral corticosteroids to self-administer Children younger than 5 years of age generally have a difficult
Section 3

at the first sign of significant deterioration.1 In addition, H.T. time coordinating the use of standard MDIs; therefore, the ICS
should have a home peak flow meter so that he can objectively and SABA should be administered by another mode of delivery.
determine the severity of his attacks. Finally, the β 2 -agonist con- For example, the as-needed β 2 -agonist and scheduled ICS could
troversy does not extend to use of high dosages in the acute-care be administered with an inhalational aid such as a spacer or
setting. High dosages are essential in the ED and hospital and, as valved holding chamber, which is connected to an MDI. A nebu-
discussed previously, usually are tolerated very well. lized corticosteroid preparation (budesonide) is available for very
Pulmonary Disorders

young children.69 Inhalation aids (also called spacers) significantly


improve the efficacy of medications that are administered by MDI
CHRONIC ASTHMA in very young children or other patients who are unable to coordi-
nate the plain inhalers correctly.70 The AeroChamber is a widely
used valved holding chamber (the medication stays in the cham-
Classification of Severity ber for a few seconds until the patient inhales slowly and the
inhalation valve opens). Studies have shown that many children
CASE 23-3 as young as 2 and 3 years of age can use MDIs with spacer devices
QUESTION 1: B.C. is a 3-year-old, 16-kg boy with a 1.5-year by modeling after a parent.71,72 Spacer devices with face masks
history of recurrent wheezing 3 days per week and noc- are required for some young children. Extender device-assisted
turnal awakenings four times per month. His medications delivery of aerosolized medications is as effective as nebulization
include albuterol syrup (2 mg/5 mL) 1 teaspoonful three in the home management of chronic severe asthma73 and even
times a day (TID) and albuterol metered-dose aerosol inhala- in the ED treatment of asthma in children.30–32,73 The InspirEase
tion QID PRN for wheezing. B.C. demonstrates the use of and AeroChamber are examples of devices that contain a flow
the inhaler with his mother’s assistance. The mother holds indicator whistle that sounds if the patient inhales rapidly. This
the inhaler in B.C.’s mouth and actuates it at the end of whistle is particularly effective in teaching the patient the appro-
a deep inhalation. B.C.’s mother tells the clinician that he priate slow inhalation technique. As a 3-year-old, B.C. may not
appears “jittery” after taking albuterol syrup. What is the need a spacer with a face mask, but the clinician should ver-
first step in deciding how to improve B.C.’s long-term drug ify correct use of the device by observation of the patient’s or
therapy? caregiver’s administration technique.
Alternatively, the ICS could be administered to selected chil-
While keeping in mind the goals of therapy defined by the dren by using a breath-activated dry powder inhaler (e.g., budes-
NIH guidelines (EPR-3),1 the first step here is to classify B.C.’s onide [Pulmicort Flexhaler] or fluticasone [combined with sal-
asthma severity (refer to Table 23-1 for infants to age 4 years). meterol in Advair Diskus]). In young children, Diskus has the
Because B.C. has symptoms 3 days per week and 4 nights per advantage of requiring a lower PIF than Flexhaler, and it has
month, he should be classified as “moderate persistent.” Note been used successfully in children as young as 4 years of age.74
that the presence of even one of the features of severity places Another option is the Twisthaler used for delivery of mometa-
the patient in that category. sone (Asmanex), but it is only approved for children at least
12 years of age.
Many pediatricians may choose a nebulizer to administer
Selection of Appropriate Initial SABAs to a 3-year-old child. This method is certainly accept-
Long-Term Therapy able and very common, but it takes longer to administer the
medication (about 15 minutes), and the device must be properly
CASE 23-3, QUESTION 2: What would be a reasonable ini- cleaned and maintained. Because B.C. recently turned 3 years
tial regimen for B.C.? old, therapy should be initiated with nebulized budesonide (Pul-
micort Respules) in a low dose of 0.25 mg BID. As mentioned
Because B.C.’s asthma is classified as moderate persistent, the previously, EPR-31 suggests an initial trial of low-dose ICS in very
clinician is now in a position to select appropriate long-term young children who have not previously been treated with ICS.
therapy. Using EPR-31 for very young children (Fig. 23-7), low- The plan is to switch B.C. to a dry powder inhaler or MDI-spacer
or medium-dose inhaled corticosteroids (ICS) with an as-needed in 12 months, as soon as he and his caregiver can demonstrate cor-
SABA would be the choice for B.C.1 EPR-3 suggests an initial rect use. B.C.’s clinician should assess the response to low-dose
trial of low-dose ICS in very young children who have not previ- budesonide in 4 weeks. Therapy can be stepped up if necessary
ously been treated with ICS. B.C. could receive ICS treatments and subsequently stepped down in the coming months, if possi-
via nebulizer (budesonide) or MDI plus a spacer. Most children ble, so that optimal asthma control is achieved at the lowest ICS
aged 3 years or younger cannot use dry powder inhalers because dose possible
P1: Trim: 8.375in × 10.875in Top: 0.373in Gutter: 0.664in
LWBK915-23 LWW-KodaKimble-educational October 27, 2011 8:22

corticosteroid therapy not only offers excellent relief of nasal 585


CASE 23-3, QUESTION 4: B.C.’s parents are wary of their
symptoms but also improves asthma control.6 Good control of
son taking corticosteroids on a continual basis after having
rhinitis is helpful in maintaining optimal asthma control1,2 (see
read about serious side effects attributed to corticosteroids
Chapter 25, Acute and Chronic Rhinitis). Despite precautions
on the Internet. How should the parents be counseled?
listed in manufacturer literature that older (sedating) antihis-
tamines should be avoided in asthma, these agents are safe in
Corticosteroids reach the systemic circulation minimally via
patients with asthma.1,2
inhalation in part because they are largely inactivated through
first-pass hepatic metabolism. However, a dose-dependent
response is evident, and clinically significant adverse events result- Corticosteroids
ing from systemic exposure can occur, albeit more commonly
with doses at the high end of the range. Long-term studies in CASE 23-5
pediatric patients have examined the effects of ICS on growth
QUESTION 1: S.T., a 12-year-old girl with severe persis-
reduction, bone density, and adrenal suppression.
tent asthma, has not been well controlled on mometasone
Although ICS can cause a mild and temporary reduction in
(Asmanex) one inhalation daily (she admits to using it only
growth velocity, final height attained in adulthood appears to

Chapter 23
when she feels as if she needs it) and uses as-needed inhaled
be within normal limits.75 Bone density and risk of fractures
albuterol MDI five or six times every day. When her symp-
have not been found in the vast majority of investigations to be
toms worsen, she uses her “breathing machine” (nebulizer)
affected by ICS.75 Although a measurable reduction in serum
at home. S.T. awakens most nights with wheezing. She has
and urinary cortisol levels is not an uncommon finding in studies
been hospitalized four times in the last 2 years and has
of ICS, clinically significant adrenal insufficiency solely caused
required “bursts” of prednisone with increasing frequency.
by ICS is rare, although again it is more likely to occur with
S.T. has missed many days of school in the past year and has

Asthma
high doses.75 In summary, these adverse effects are generally of
withdrawn from physical education classes and her extracur-
minimal clinical significance, with the benefits of well-controlled
ricular sports activity after school. Her parents are con-
asthma far outweighing the risks.
cerned about her increased use of prednisone now that she
The most common local side effect with ICS therapy is
is approaching puberty. S.T. is just finishing a 2-week course
oropharyngeal candidiasis (thrush), but this problem is rare with
of prednisone 20 mg/day and has a round facies appear-
any delivery system. With MDIs, it can be further minimized
ance typical of chronic oral corticosteroid use. On physical
by use of a spacer device. Rinsing the mouth with water after
examination, S.T. has diffuse expiratory wheezes, and pul-
use of any ICS is also recommended. Another possible local side
monary function testing reveals significant reversibility. Her
effect is hoarseness (dysphonia), and spacers may not effectively
FEV1 is only 60% predicted before use of albuterol in the
reduce this problem.76 Dry powder devices (e.g., Flexhaler) may
physician’s office and improves to 75% predicted 15 min-
have less dysphonia associated with their use, but further study
utes after use of the SABA. What actions are needed to
is required to verify this.76
improve S.T.’s care?

Seasonal Asthma Because S.T. is suffering needlessly and requiring frequent


systemic corticosteroids, all efforts must be made to optimize
CASE 23-4 other therapies and to minimize systemic corticosteroid tox-
QUESTION 1: C.V., a 33-year-old woman, presents to the icities. Although S.T. is receiving an ICS, she admits to poor
clinic with a history of asthma and seasonal allergic rhinitis adherence. Therefore, with her severe persistent asthma, she
(“hay fever”) each spring but not the rest of the year. She initially needs higher-dose ICS therapy. Per EPR-3,1 she should
describes her asthma as “mild” and intermittent. Except also receive a LABA. Although short bursts of prednisone (e.g.,
during springtime, her daytime symptoms occur less than 40 mg/day for 3 days) are very helpful occasionally, frequent short
once per week, and she does not have nocturnal symp- courses often indicate the need to optimize other therapies. Some
toms. Each spring, however, these symptoms worsen, and patients require courses of 1 to 2 weeks. S.T. is requiring longer
she requires her albuterol inhaler (her only asthma medi- frequent bursts and is showing signs of adverse effects. Obviously,
cation) TID or QID every day. During springtime, she takes S.T. and her parents also need a concerted and persistent effort
a nonprescription antihistamine, which offers some relief. in patient education as a partnership (refer to Patient Education
How can C.V.’s management be improved? and Outcomes sections).
ICS are chemically modified to maximize topical effective-
C.V. appears to have intermittent asthma during most of the ness while minimizing systemic toxicities. Tables 23-6 and 23-7
year but converts to moderate persistent asthma combined with are taken from EPR-31 and compare the dosages of ICS prod-
worsening rhinitis symptoms in the springtime. This syndrome ucts. These differences in dosages (low, medium, and high)
is consistent with a diagnosis of seasonal asthma and allergic reflect differences among ICS in receptor-binding affinity and
rhinitis. Although as-needed albuterol is appropriate most of the topical potency. There are also differences among these agents
year for C.V., she needs anti-inflammatory therapy during the in oral bioavailability (i.e., absorption of drug that is swallowed
spring.1 Therapy to reduce airway inflammation should begin after inhalation) and systemic availability via absorption from
before the onset of tree and grass pollen season and continue the lungs. Of these two variables, absorption from the lungs is
throughout the spring (e.g., 3 months). Per the NIH guidelines, the most likely contributor to possible hypothalamic-pituitary-
an excellent treatment option for C.V. is low-dose ICS combined adrenal (HPA) suppression or other systemic effects. Fortunately,
with a long-acting inhaled β 2 -agonist (LABA). Monotherapy with the total absorption has not been shown to be clinically impor-
medium-dose ICS would also be an acceptable therapy. Not only tant except at the higher recommended dosages. Although the
are the causes and pathophysiology of allergic rhinitis and allergic various ICS are not equipotent on a microgram-per-microgram
asthma similar, poorly controlled rhinitis also serves as a major basis, major differences in efficacy or adverse effects are not
asthma trigger. In addition, C.V. should also ideally receive an firmly established.1,75 For patients with severe persistent asthma,
intranasal corticosteroid if antihistamines (preferably nonsedat- a logical choice would be a high-potency agent that would
ing) do not provide optimal relief of her allergic rhinitis. Intranasal allow for a minimum number of inhalations per day, potentially
P1: Trim: 8.375in × 10.875in Top: 0.373in Gutter: 0.664in
LWBK915-23 LWW-KodaKimble-educational October 27, 2011 8:22

586
TA B L E 2 3 - 6
Estimated Comparative Daily Dosages for Inhaled Corticosteroids in Children

Low Daily Dose Medium Daily Dose High Daily Dose


Child Child Child Child Child Child
Drug 0–4 Years 5–11 Years 0–4 Years 5–11 Years 0–4 Years 5–11 Years

Beclomethasone HFA
40 or 80 mcg/puff NA 80–160 mcg NA >160–320 mcg NA >320 mcg
Budesonide DPI
90, 180, or 200 mcg/inhalation NA 180–400 mcg NA >400–800 mcg NA >800 mcg
Budesonide inhaled
Inhalation suspension for nebulization 0.25–0.5 mg 0.5 mg >0.5–1 mg 1 mg >1 mg 2 mg
Flunisolide
250 mcg/puff NA 500–750 mcg NA 1,000–1,250 mcg NA >1,250 mcg
Flunisolide HFA
Section 3

80 mcg/puff NA 160 mcg NA 320 mcg NA >640 mcg


Fluticasone
HFA/MDI: 44, 110, or 230 mcg/puff 176 mcg 88–176 mcg >176–352 mcg >176–352 mcg >352 mcg >352 mcg
DPI: 50, 100, or 250 mcg/inhalation NA 100–200 mcg NA >200–400 mcg NA >400 mcg
Mometasone DPI
Pulmonary Disorders

200 mcg/inhalation NA NA NA NA NA NA
Triamcinolone acetonide
75 mcg/puff NA 300–600 mcg NA >600–900 mcg NA >900 mcg

DPI, dry powder inhaler; HFA, hydrofluoroalkane; MDI, metered-dose inhaler; NA, not approved and no data available for this age group.
Reprinted from National Institutes of Health. Expert Panel Report 3: Guidelines for the Diagnosis and Management of Asthma. Bethesda, MD: National Heart, Lung, and Blood
Institute; 2007. NIH publication 07-4051.

improving treatment adherence. Furthermore, the delivery suppression.75 The clinical significance of this suppression has
system used affects pulmonary deposition. For example, the yet to be firmly established.
Turbuhaler (now available as the Flexhaler) delivers about twice Although low to moderate dosages of ICS are accepted as
the dose of budesonide versus an MDI and is associated with being quite safe, very high dosages continue to be scrutinized
excellent efficacy if used correctly.77 Clinical trials evaluating flu- regarding the potential adverse effects.1 Clearly, for patients who
ticasone efficacy administered via the Diskus have likewise shown require high dosages for optimal control of asthma, the bene-
excellent efficacy.78 The differences among the various dry pow- fits of therapy with these agents far outweigh the risks.1,75 A
der inhalers are discussed later in this chapter. Addition of a spacer possible association between prolonged, very high dosages of
device to an MDI also enhances pulmonary deposition. In very ICS and cataracts79,80 and glaucoma81 has been reported. EPR-3
high dosages (the equivalent of 1,600 mcg/day of beclometha- (Table 23-8) has summarized recent research that has allayed con-
sone dipropionate), all ICS produce some degree of HPA-axis cerns regarding use of ICS therapy and growth suppression in

TA B L E 2 3 - 7
Estimated Comparative Daily Dosages for Inhaled Corticosteroids for Youths ≥12 Years of Age and Adults

Drug Low Daily Dose Adult Medium Daily Dose Adult High Daily Dose Adult

Beclomethasone HFA
40 or 80 mcg/puff 80–240 mcg >240–480 mcg >480 mcg
Budesonide DPI
90, 180, or 200 mcg/inhalation 180–600 mcg >600–1,200 mcg >1,200 mcg
Flunisolide
250 mcg/puff 500–1,000 mcg >1,000–2,000 mcg >2,000 mcg
Flunisolide HFA
80 mcg/puff 320 mcg >320–640 mcg >640 mcg
Fluticasone
HFA/MDI: 44, 110, or 220 mcg/puff 88–264 mcg >264–440 mcg >440 mcg
DPI: 50, 100, or 250 mcg/inhalation 100–300 mcg >300–500 mcg >500 mcg
Mometasone DPI
200 mcg/inhalation 200 mcg 400 mcg >400 mcg
Triamcinolone acetonide
75 mcg/puff 300–750 mcg >750–1,500 mcg >1,500 mcg

DPI, dry powder inhaler; HFA, hydrofluoroalkane: MDI, metered-dose inhaler.


Reprinted from National Institutes of Health. Expert Panel Report 3: Guidelines for the Diagnosis and Management of Asthma. Bethesda, MD: National Heart, Lung, and Blood
Institute; 2007. NIH publication 07-4051.
P1: Trim: 8.375in × 10.875in Top: 0.373in Gutter: 0.664in
LWBK915-23 LWW-KodaKimble-educational October 27, 2011 8:22

587
TA B L E 2 3 - 8
Long-Term Control Medications

Name/Products
(Listed Alphabetically) Indications/Mechanisms Potential Adverse Effects Therapeutic Issues (Not All Inclusive)

Corticosteroids
(Glucocorticoids)
Inhaled (ICS): Indications
s s s
Beclomethasone Long-term prevention of Cough, dysphonia, oral thrush Spacer/holding chamber devices with
Budesonide symptoms: suppression, control, (candidiasis). nonbreath-activated MDIs and mouth
s
Ciclesonide and reversal of inflammation. In high doses (Tables 23-6, 23-7), washing after inhalation decrease local
s systemic effects may occur, side effects.
Flunisolide Reduce need for oral
s
Fluticasone corticosteroid. although studies are not Preparations are not absolutely
Mometasone Mechanisms conclusive, and clinical interchangeable on a mcg or per puff
s significance of these effects has basis (Tables 23-6, 23-7 for estimated
Anti-inflammatory. Block late
reaction to allergen and reduce not been established (e.g., clinical comparability). New delivery

Chapter 23
airway hyperresponsiveness. adrenal suppression, devices may provide greater delivery to
Inhibit cytokine production, osteoporosis, skin thinning, and airways; this change may affect dose.
s
adhesion protein activation, and easy bruising). In low to medium The risks of uncontrolled asthma should
inflammatory cell migration and doses, suppression of growth be weighed against the limited risks of
activation. velocity has been observed in ICS therapy. The potential but small risk
s
Reverse β 2 -receptor children, but this effect may be of adverse events is well balanced by their
downregulation. Inhibit transient, and the clinical efficacy.

Asthma
significance has not been s
microvascular leakage. “Adjustable dose” approach to treatment
established. may enable reduction in cumulative dose
of ICS treatment with time without sacri-
ficing maintenance of asthma control.
s
Dexamethasone is not included as an ICS
for long-term control because it is highly
absorbed and has long-term suppressive
side effects.
Systemic: Indications
Methylprednisolone s s s
For short-term (3–10 days) Short-term use: Reversible Use at lowest effective dose. For
Prednisolone “burst”: to gain prompt control of abnormalities in glucose long-term use, alternate-day am dosing
Prednisone inadequately controlled persistent metabolism, increased appetite, produces the least toxicity. If daily doses
asthma. fluid retention, weight gain, are required, one study shows improved
s
For long-term prevention of mood alteration, hypertension, efficacy with no increase in adrenal
symptoms in severe persistent peptic ulcer, and rarely aseptic suppression when administered at 3 pm
asthma, suppression, control, and necrosis. rather than in the morning.
s
reversal of inflammation. Long-term use: Adrenal axis
Mechanisms suppression, growth
s
Same as inhaled. suppression, dermal thinning,
hypertension, diabetes, Cushing
syndrome, cataracts, muscle
weakness; in rare instances,
impaired immune function.
s
Consideration should be given
to coexisting conditions that
could be worsened by systemic
corticosteroids, such as herpes
virus infections, varicella,
tuberculosis, hypertension,
peptic ulcer, diabetes mellitus,
osteoporosis, and Strongyloides.
Immunomodulators
Omalizumab (anti-IgE) Indications
s s s
For subcutaneous use Long-term control and prevention Pain and bruising of injection Monitor patients after injection. Be
of symptoms in adults (≥12 years sites has been reported in prepared and equipped to identify and
old) who have moderate or severe 5%–20% of patients. treat anaphylaxis that may occur.
s s
persistent allergic asthma Anaphylaxis has been reported in The dose is administered either every
inadequately controlled with ICS. 0.2% percent of treated patients. 2 or 4 weeks and is dependent on the
s
Mechanisms Malignant neoplasms were patient’s body weight and IgE level before
s
Binds to circulating IgE, reported in 0.5% of patients therapy.
s
preventing it from binding to the compared with 0.2% receiving A maximum of 150 mg can be
high-affinity (FCE RI) receptors on placebo; relationship to drug is administered in one injection.
s
basophils and mast cells. unclear. Needs to be stored under refrigeration at
s
Decreases mast cell mediator 2◦ –8◦ C.
s
release from allergen exposure. Whether patients will develop significant
s
Decreases the number of FCE RIs antibody titers to the drug with
in basophils and submucosal cells. long-term administration is unknown.
(continued)
P1: Trim: 8.375in × 10.875in Top: 0.373in Gutter: 0.664in
LWBK915-23 LWW-KodaKimble-educational October 27, 2011 8:22

588
TA B L E 2 3 - 8
Long-Term Control Medications (Continued )

Name/Products
(Listed Alphabetically) Indications/Mechanisms Potential Adverse Effects Therapeutic Issues (Not All Inclusive)

Leukotriene
Receptor
Antagonists
Montelukast tablets Mechanisms
s s
and granules Leukotriene receptor antagonist; May attenuate EIB in some patients, but
selective competitive inhibitor of less effective than ICS therapy.
s
CysLT1 receptor. Do not use LTRA + LABA as a substitute
for ICS + LABA.
Indications
s s s
Long-term control and No specific adverse effects have A flat dose–response curve, without
prevention of symptoms in mild been identified. further benefit if dose is increased above
s
Section 3

persistent asthma for patients Rare cases of Churg-Strauss those recommended.


≥1 year of age. May also be used have occurred, but the
with ICS as combination therapy association is unclear.
in moderate persistent asthma.
s s s
Zafirlukast tablets Long-term control and Postmarketing surveillance has Administration with meals decreases
prevention of symptoms in mild reported cases of reversible bioavailability; take at least 1 hour before
persistent asthma for patients hepatitis and, rarely, or 2 hours after meals.
Pulmonary Disorders

s
≥7 years of age. May also be used irreversible hepatic failure Zafirlukast is a microsomal P-450 enzyme
with ICS as combination therapy resulting in death and liver inhibitor that can inhibit the metabolism
in moderate persistent asthma. transplantation. of warfarin. INRs should be monitored
during coadministration.
s
Patients should be warned to discontinue
use if they experience signs and symptoms
of liver dysfunction (right upper quadrant
pain, pruritus, lethargy, jaundice, nausea),
and serum ALTs should be monitored.
5-Lipoxygenase
Inhibitor
Zileuton tablets Mechanisms
s
Inhibits the production of
leukotrienes from arachidonic
acid, both LTB4 and the cysteinyl
leukotrienes.
Indications
s s s
Long-term control and Elevation of liver enzymes has Zileuton is microsomal P-450 enzyme
prevention of symptoms in mild been reported. Limited case inhibitor that can inhibit the metabolism
persistent asthma for patients reports of reversible hepatitis of warfarin and theophylline. Doses of
≥12 years of age. and hyperbilirubinemia. these drugs should be monitored
s
May be used with ICS as accordingly.
s
combination therapy in Monitor hepatic enzymes (ALT).
moderate persistent asthma in
patients ≥12 years of age.
Long-Acting
β2 -Agonists
(LABA) Indications s s
s Tachycardia, skeletal muscle Not to be used to treat acute symptoms
Inhaled LABA: Long-term prevention of
tremor, hypokalemia, or exacerbations.
Formoterol symptoms added to ICS. s
s prolongation of QTc interval in Should not be used as monotherapy for
Salmeterol Prevention of EIB.
s overdose. long-term control of asthma or as
Not to be used to treat acute s
A diminished anti-inflammatory therapy.
symptoms or exacerbations. s
bronchoprotective effect may May provide more effective symptom
Mechanisms
s occur within 1 week of chronic control when added to standard doses of
Bronchodilation. Smooth
therapy. Clinical significance ICS compared with increasing the ICS
muscle relaxation after adenylate
has not been established. dosage.
cyclase activation and increase in s s
Potential risk of uncommon, Clinical significance of potentially
cyclic AMP, producing functional
severe, life-threatening or fatal developing tolerance is uncertain because
antagonism of bronchoconstric-
exacerbation: see text for studies show that symptom control and
tion.
s additional discussion regarding bronchodilation are maintained.
Compared with SABA, s
safety of LABAs. Decreased duration of protection against
salmeterol (but not formoterol)
EIB may occur with regular use.
has slower onset of action
(15–30 minutes). Both salmeterol
and formoterol have longer
duration (>12 hours) compared
with SABA.
(continued)
P1: Trim: 8.375in × 10.875in Top: 0.373in Gutter: 0.664in
LWBK915-23 LWW-KodaKimble-educational October 27, 2011 8:22

589
TA B L E 2 3 - 8
Long-Term Control Medications (Continued )

Name/Products
(Listed Alphabetically) Indications/Mechanisms Potential Adverse Effects Therapeutic Issues (Not All Inclusive)

Oral:
s
Albuterol, Inhaled route is preferred because
sustained-release LABAs are longer acting and have fewer
side effects than oral sustained-release
agents. Oral agents have not been
adequately studied as adjunctive
therapy with ICS.
Methylxanthines
Theophylline, Indications
s s s
sustained-release Long-term control and Dose-related acute toxicities Maintain steady-state serum concen-
tablets and capsules prevention of symptoms in mild include tachycardia, nausea and trations between 5 and 15 mcg/mL.

Chapter 23
persistent asthma or as vomiting, tachyarrhythmias Routine serum concentration
adjunctive with ICS in moderate (SVT), central nervous system monitoring is essential owing to
or persistent asthma. stimulation, headache, significant toxicities, narrow
seizures, hematemesis, therapeutic range, and individual
Mechanisms hyperglycemia, and differences in metabolic clearance.
s
Bronchodilation. Smooth muscle hypokalemia. Absorption and metabolism may be

Asthma
relaxation from affected by numerous factors that can
s
phosphodiesterase inhibition and Adverse effects at usual produce significant changes in
possibly adenosine antagonism. therapeutic doses include steady-state serum theophylline
s
May affect eosinophilic insomnia, gastric upset, concentrations.
s
infiltration into bronchial aggravation of ulcer or reflux, Patients should be told to discontinue if
mucosa as well as decreases increase in hyperactivity in they experience toxicity.
s
T-lymphocyte numbers in some children, and difficulty in Not generally recommended for
epithelium. urination in elderly men who exacerbations. There is minimal
s
Increases diaphragm have prostatism. evidence for added benefit to optimal
contractility and mucociliary doses of SABA. Serum concentration
clearance. monitoring is mandatory.

ALT, alanine aminotransferase; AMP, adenosine monophosphate; Anti-IgE, anti-immunoglobulin E; EIB, exercise-induced bronchospasm; FCE RI, high-affinity IgE receptor;
ICS, inhaled corticosteroids; IGS, inhaled glucocorticoids; INR, international normalized ratio; LABA, long-acting inhaled β 2 -agonist; LTB4 , leukotriene B4 ; LTRA,
leukotriene receptor agonist; MDI, metered-dose inhaler; SABA, short-acting inhaled β 2 -agonist; SVT, supraventricular tachycardia.
Reprinted from National Institutes of Health. Expert Panel Report 3: Guidelines for the Diagnosis and Management of Asthma. Bethesda, MD: National Heart, Lung, and Blood
Institute; 2007. NIH publication 07-4051.

children (i.e., the decrease in growth velocity is small and not combination with a LABA (Fig. 23-9, Table 23-3). More aggres-
progressive, and appears to be reversible).1,82,83 sive therapy initially is especially important in S.T. because of her
four hospitalizations in the last 2 years. In partnership with S.T.
CASE 23-5, QUESTION 2: How should S.T. be managed? and her parents, her preference should be determined as to the
delivery method (i.e., discuss options with her regarding breath-
In the treatment of S.T., most clinicians would begin with a activated devices or MDI and spacer, including which spacer).
short course (e.g., 1 week) of systemic corticosteroids to max- Ideally, the clinician should recognize her emerging indepen-
imally improve her pulmonary function. This approach is con- dence as a 12-year-old and talk with her alone and then with her
sistent with EPR-3,1 which recommends gaining quick control. parents. After S.T. is stabilized for 3 months, attempts should be
Using short-course systemic therapy is logical because it is inex- made to slowly decrease the ICS dosage every 3 months until
pensive, efficacious, and associated with low risk. While gaining the lowest effective dosage is achieved. Administration of the
quick control with a short course of oral corticosteroids, it is log- total daily ICS dose is preferred twice daily or, in many patients
ical to start ICS in many patients at a low1,84 to moderate dosage with mild to moderate persistent asthma, once daily.1 Because
(as defined in Tables 23-6 and 23-7). The evidence that ICS are adherence is a major determinant of success or failure with ICS
highly effective in persistent asthma is unequivocal. Patients in and other therapies, simplified regimens and continued patient
this category who consistently take ICS are at a lower risk of hos- education and contact are essential.1
pitalization and death, and one study found an increased risk of
death after discontinuation of ICS compared with patients who COMBINATION OF INHALED CORTICOSTEROIDS
remained on these drugs.85,86 ICS therapy should be initiated AND LONG-ACTING INHALED β2 -AGONISTS
concomitantly with a short course of systemic corticosteroid
therapy. Patient education at this time may have greater effec- CASE 23-5, QUESTION 3: Because S.T. is 12 years old, what
tiveness because some patients are more attentive after having options are appropriate to minimize the ICS dosage, realiz-
just experienced an exacerbation, and they know that change is ing that aggressive therapy is needed? What are the risks
needed to improve their health. Similar to adults, intensification associated with LABAs? Clinicians should monitor for which
of therapy with increasing the ICS dose, adding montelukast, or local side effects in S.T. with ICS therapy?
adding a LABA was found to be successful in pediatric patients,
with the LABA being the most likely addition to be beneficial.87 LABAs have been very successful in prevention of “step-
Because S.T.’s asthma is classified as severe persistent, it is rea- ping up” the dose of ICS while markedly enhancing overall
sonable to start her on a moderate to high dosage of an ICS in asthma control,77,78,88–90 and this fact is reflected in EPR-3.1 A
P1: Trim: 8.375in × 10.875in Top: 0.373in Gutter: 0.664in
LWBK915-23 LWW-KodaKimble-educational October 27, 2011 8:22

590 reasonable therapeutic option for S.T. would be the combina- mally proceed at a slower pace. However, fluticasone 500 mcg
tion of fluticasone 500 mcg and salmeterol 50 mcg via Diskus daily, especially in combination with salmeterol, will likely be
(Advair 500) 1 inhalation BID, the budesonide/formoterol (Sym- an adequate dose for S.T. After S.T.’s dose of fluticasone was
bicort) combination inhaler (160 mcg/4.5 mcg; 2 inhalations stepped down to 500 mcg/day, she started requiring only slightly
BID), or the mometasone/formoterol (Dulera) combination more PRN albuterol (but still was symptom free most days).
inhaler (200 mcg/5 mcg; 2 inhalations BID). S.T. should be
re-evaluated in 2 weeks. The plan is to step down the dose of
CASE 23-5, QUESTION 6: If symptoms recur during step-
fluticasone after excellent asthma control is achieved.
down, what management could possibly facilitate dosage
reduction of the ICS in S.T.?
ADVERSE EFFECTS ASSOCIATED WITH LONG-ACTING
INHALED β2 -AGONISTS
During follow-up evaluations, clinicians should carefully
investigate factors that may contribute to poor asthma con-
CASE 23-5, QUESTION 4: S.T.’s parents recently discov-
trol, including exposure to inhalant allergens, indoor or outdoor
ered an article in a national newspaper that addressed con-
irritants, medications, and tobacco smoke. Secondhand smoke
cerns of using LABAs because of an increased risk of death.
exposure has been demonstrated to negate the benefit of ICS in
Because S.T.’s parents have called and left a message for
Section 3

children, and asthma patients who smoke have reduced response


the clinician, what would be important information to share
to ICS therapy.96,97
with her family regarding this issue? What other side effects
should S.T. and her family be aware of?

Several randomized trials for more than a decade have demon- Leukotriene Modifiers
strated that LABAs have minimal adverse effects (e.g., tachycar-
Pulmonary Disorders

CASE 23-6
dia, tremor).1,77,78,88–90 Although there are limited data, based
primarily on one study (SMART),91 there may be a very small QUESTION 1: P.W. is a 52-year-old man with mild persistent
increased risk of asthma-related death and asthma exacerbation asthma. His asthma symptoms began when he was 2 years
in patients receiving LABAs. This small risk is likely attributable of age, and he has never smoked. P.W. has had numerous
to patients receiving LABAs without concomitant ICS. In fact, drug regimens for his asthma during the years, but he tells
EPR-31 recommends against the use of LABAs as monotherapy his physician that he wants the simplest regimen possible
for long-term control of persistent asthma. SMART91 data sug- and that he prefers oral medication if at all possible. What
gested that black patients may be at greater risk, but much more is a good choice for controller therapy in P.W.?
study is needed to confirm these data. LABA therapy should only
be used in combination with ICS in patients with asthma. Com- In patients of any age with mild persistent asthma, and
bination ICS/LABA therapy for asthma is safe and effective.1,92 certainly in children or adolescents, an oral agent such as mon-
EPR-31 suggests giving equal consideration to a moderate dose telukast with once-daily dosing at bedtime (or zafirlukast with
of ICS alone or low-dose ICS combined with LABA for patients twice-daily dosing) has obvious advantages. Zileuton is another
with moderate persistent asthma. In 2010, the US Food and Drug leukotriene modifier, but it has more drug interactions and
Administration (FDA) offered recommendations regarding the adverse effects than the other available agents. An ICS is the
use of LABA in patients with asthma, including always using com- preferred treatment for mild persistent asthma. Studies com-
bination products (ICS/LABA) versus two single-agent products paring ICS with leukotriene receptor agonists have consistently
to help ensure LABA are never used alone in asthma patients.93 demonstrated the superiority of ICS for most asthma outcome
In response to the FDA, editorials by EPR-3 members and others measures,1 but in patients (adults or children) who much prefer
question some aspects of the FDA recommendations.94,95 For oral therapy to inhaling medication every day, leukotriene mod-
S.T., who has severe persistent asthma, ICS/LABA combination ifiers are a reasonable option. The simplest and safest possible
therapy is preferred.1 oral controller regimen for P.W. is montelukast 10 mg at bedtime
(with PRN inhaled albuterol). Bedtime dosing with montelukast
STEP-DOWN TREATMENT is recommended because it will have peak activity late at night
and in the early morning hours, when asthma symptoms tend to
CASE 23-5, QUESTION 5: After being adherent to her be more frequent. It is likely that this regimen will result in very
new therapy (fluticasone 500 mcg/salmeterol 50 mcg BID) good asthma control. If P.W. does not have good control of his
for 1 month, S.T.’s asthma control has markedly improved. asthma when he returns to the clinic in a few weeks, switching
She required no ED visits or hospitalizations, was sleeping to a low-dose ICS in the evening only would keep the regimen
through the night, and began to exercise again. Her per- simple while enhancing efficacy.
sonal best PEF was 320 L /minute, and she was staying in her
green zone (260–320 L /minute), requiring PRN albuterol no
more than once per week. After 2 more months, S.T. con- Theophylline
tinues to do well. Although S.T. clearly needs long-term ICS
DOSING
therapy, after 3 months of excellent response, her clinician
is now ready to step down from high-dose fluticasone. What CASE 23-7
is a prudent approach to dosage reduction? QUESTION 1: K.J., a 14-year-old, 40-kg girl, has a history of
recurring cough and wheezing. These symptoms worsen on
1
EPR-3 suggests stepping down the dose at a rate of 25% to vigorous running or when she has an upper respiratory infec-
50% every 3 months to the lowest dose that maintains control. A tion. She has not required hospitalization for these symp-
step down to fluticasone 250 mcg/salmeterol 50 mcg (Advair 250) toms but has missed a few school days. She has symptoms
BID would be a reasonable reduction in therapy for S.T. If a single daily and uses her pirbuterol inhaler more than two times
ICS product had been started in S.T. initially (e.g., budesonide, daily. K.J. has a family history of asthma. A diagnosis of
beclomethasone, fluticasone), the dosage reduction would nor-
P1: Trim: 8.375in × 10.875in Top: 0.373in Gutter: 0.664in
LWBK915-23 LWW-KodaKimble-educational October 27, 2011 8:22

591
TA B L E 2 3 - 9 TA B L E 2 3 - 1 0
Theophylline Dosing Guide for Chronic Usea,b Food and Drug Administration Guidelines for
Theophylline Dosing in Infantsa
Starting dose for children 1–15 years <45 kg: 12–14 mg/kg/day to
maximum of 300 mg/day Premature Neonates
Starting dose for adults and children 1–15 years >45 kg: 300 mg/day
Titrate dose upward after 3 days if necessary and if tolerated to: <24 days postnatal age: 1.0 mg/kg every 12 hours
s
16 mg/kg/day to maximum of 400 mg/day in children 1–15 years ≥24 days postnatal age: 1.5 mg/kg every 12 hours
<45 kg Term Infants and Infants Up to 52 Weeks of Age
s
400 mg/day in adults and children >45 kg
Titrate dose upward after 3 more days if necessary and if tolerated to: Total daily dose (mg) = [(0.2 × age in weeks) + 5.0] × (kg body weight)
s s
20 mg/kg/day to a maximum of 600 mg/day in children 1–15 years Up to age 26 weeks; divide dose into 3 equal amounts
<45 kg administered at 8-hour intervals
s
s
600 mg/day in adults and in children >45 kg >26 weeks of age; divide dose into 4 equal amounts administered
at 6-hour intervals
a
Dose using ideal body weight or actual body weight, whichever is less. These
a
dosages do not apply if liver disease, heart failure, or other factors documented Final doses adjusted to a peak steady-state serum theophylline concentration of

Chapter 23
to affect theophylline clearance are present. Doses must be guided by 5–10 mcg/mL in neonates and 10–15 mcg/mL in older infants.
monitoring serum concentrations to ensure optimal safety and efficacy. Source: Hendeles L et al. Revised FDA labeling guideline for theophylline oral
b
Dosing schedule dependent on product selected; sustained-release products are dosage forms. Pharmacotherapy. 1995;15:409.
much preferred, if at all possible.
Source: Hendeles L et al. Revised FDA labeling guideline for theophylline oral
dosage forms. Pharmacotherapy. 1995;15:409.
Accordingly, the initial dosage in K.J. would be 300 mg/day in
divided doses (i.e., 150 mg every 12 hours). If tolerated, the dosage

Asthma
is increased at 3-day intervals by about 25% to the mean dose
moderate persistent asthma is made. How should K.J. be that usually is needed to produce a peak theophylline serum
managed? concentration between 5 and 10 mcg/mL. The final dosage can
be adjusted by using the guidelines listed in Table 23-11. Serum
Because K.J. has moderate persistent asthma, treatment with theophylline concentrations should be obtained at steady state
an anti-inflammatory agent is indicated. Low to moderate dose (i.e., when there have been no missed doses and no extra doses
ICS in combination with a LABA is the preferred treatment in have been taken for at least 48 hours).
a 14-year-old child with moderate persistent asthma.1 However,
not all health care professionals follow the latest evidence-based TOXICITY
guidelines, and K.J.’s clinician opts to prescribe theophylline in
combination with a low dose of budesonide via the Flexhaler. CASE 23-7, QUESTION 3: K.J. now complains of headache
and difficulty in getting to sleep. Why should a theophylline
CASE 23-7, QUESTION 2: What dosage of theophylline is serum concentration be evaluated?
appropriate for K.J.?
Theophylline side effects can be related to excessive serum
Low doses of budesonide combined with twice-daily theo- concentrations, or adverse effects can be transient and unrelated
phylline that resulted in a median serum theophylline concentra- to the amount in serum. Unfortunately, it is not always pos-
tion of 8.7 mcg/mL was superior in efficacy to high-dose budes- sible to determine which it might be. Side effects can include
onide as single controller therapy.98 Thus, it is wise to give a headache, nausea, vomiting, irritability or hyperactivity, insom-
therapeutic trial with low-dose theophylline initially, aiming for nia, and diarrhea. With higher serum theophylline levels, cardiac
serum concentrations between 5 and 10 mcg/mL. In the non- arrhythmias, seizures, and death can occur.99 Less severe symp-
acute asthma patient in whom the theophylline dose require- toms may not be present before the onset of cardiac arrhythmias
ment is unknown, dosages suggested for ages older than 1 year of or seizures and cannot be relied on as a forewarning of these
age are listed in Table 23-9; infant dosing is listed in Table 23-10. more serious adverse theophylline effects. It is important not to

TA B L E 2 3 - 1 1
Adjusting Doses of Theophylline Based on Serum Concentrations

Peak Theophylline
Concentration Approximate Adjustment
(mcg/mL)a in Daily Dose Comment

<5.0 ↑ by 25% Recheck serum theophylline concentration.


5–10 ↑ by 25% if clinically indicated Recheck serum concentration; ↑ dose only if poor response to therapy.
10–12 Cautious 10% ↑ if clinically indicated If asymptomatic, no ↑ needed. Recheck serum theophylline concentration
before further dose changes.
12–15 Occasional intolerance requires a 10% ↓ If asymptomatic, no dose change needed unless side effects present.
16–20 ↓ by 10%–25% Even if asymptomatic and side effects absent, a dose ↓ is prudent.
20–24.9 ↓ by 50% Omit 1 dose even if asymptomatic and side effects absent, a dose ↓ is indicated.
25–29.9 ↓ by >50% Omit next doses even if asymptomatic and side effects absent; a dose ↓
indicated; repeat serum theophylline concentration after dose adjustment.
>30 Omit next doses; ↓ by 60%–75% Seek medical attention and consult regional poison center even if not
symptomatic; if >60 years of age, anticipate need for treatment of seizures.
a
It is important that levels are obtained at steady state. If laboratory results appear questionable, suggest repeat measurements.
P1: Trim: 8.375in × 10.875in Top: 0.373in Gutter: 0.664in
LWBK915-23 LWW-KodaKimble-educational October 27, 2011 8:22

592
TA B L E 2 3 - 1 2
Factors Affecting Serum Theophylline Concentrationsa

Decreases Theophylline Increases Theophylline


Factor Concentrations Concentrations Recommended Action

Food ↓ or delays absorption of some ↑ rate of absorption Select theophylline preparation that is not
sustained-release theophylline (fatty foods) affected by food.
(SR) products
Diet ↑ metabolism (high protein) ↓ metabolism (high Inform patients that major changes in diet are
carbohydrate) not recommended while taking
theophylline.
Systemic, febrile viral illness ↓ metabolism Decrease theophylline dose according to
(e.g., influenza) serum concentration level. Decrease dose
by 50% if serum concentration
measurement is not available.
Hypoxia, cor pulmonale, and ↓ metabolism Decrease dose according to serum
Section 3

decompensated congestive concentration level.


heart failure, cirrhosis
Age ↑ metabolism (1–9 years) ↓ metabolism Adjust dose according to serum
(<6 months, elderly) concentration level.
Phenobarbital, phenytoin, ↑ metabolism Increase dose according to serum
carbamazepine concentration level.
Cimetidine ↓ metabolism Use alternative histamine H2 -antagonist
Pulmonary Disorders

(e.g., famotidine or ranitidine).


Macrolides: TAO, erythromycin, ↓ metabolism Use alternative antibiotic or adjust
clarithromycin theophylline dose.
Quinolones: ciprofloxacin, ↓ metabolism Use alternative antibiotic or adjust
enoxacin, pefloxacin theophylline dose.
Rifampin ↑ metabolism Increase dose according to serum
concentration level.
Ticlopidine ↓ metabolism Decrease dose according to serum
concentration level.
Smoking ↑ metabolism Advise patient to stop smoking; increase dose
according to serum concentration level.
a
This list is not all-inclusive; for discussion of other factors, see package inserts.
TAO, troleandomycin.
Modified from National Institutes of Health. Expert Panel Report 3: Guidelines for the Diagnosis and Management of Asthma. Bethesda, MD: National Heart, Lung, and Blood
Institute; 2007. NIH publication 07-4051.

ignore any symptom consistent with theophylline toxicity. The should be screened carefully for potential interactions. As with
insomnia and headaches experienced by K.J. may not be associ- any drug interaction, the mechanism, time course, management,
ated with excessive (i.e., out of the usual therapeutic range) serum and clinical significance should be assessed before any interven-
theophylline concentrations, but a reduction in dosage should be tions. For example, cimetidine decreases theophylline clearance
contemplated because some patients experience toxicity when within 24 hours, and this interaction should be circumvented by
serum theophylline concentrations are within the therapeutic using another histamine H2 -blocker or a proton-pump inhibitor
range. Guidelines for managing toxicity have been revised.99 (Table 23-12). Classic inducers (e.g., rifampin) of cytochrome
P-450 also affect theophylline clearance, and patients should be
DRUG INTERACTIONS monitored for decreased serum concentrations. In T.R., the inter-
CASE 23-8
action with clarithromycin can easily be circumvented by using
azithromycin, which does not affect theophylline metabolism.
QUESTION 1: T.R., a 55-year-old woman with asthma, is
well controlled on theophylline SR 300 mg BID, albuterol
two puffs QID PRN, and mometasone (Asmanex) one inhala- Anticholinergics
tion at bedtime. A peak theophylline serum concentration
obtained 3 months ago was 14 mcg/mL. Six months ago, on CASE 23-9
the same dose of theophylline, her serum concentration was QUESTION 1: R.K. is a 24-year-old African American grad-
15 mcg/mL. T.R. presents with an upper respiratory tract uate medical student with moderate persistent asthma,
infection, and clarithromycin 500 mg BID is prescribed. Is which has been well controlled for 10 years with ICS ther-
this antibiotic appropriate? apy (beclomethasone HFA 80 mcg twice daily) and albuterol
PRN. Recently, he has noticed that his asthma symptoms
A large number of medications inhibit cytochrome P-450 have worsened and he is requiring his albuterol inhaler three
(CYP) isoenzymes and are capable of inhibiting the metabolism or four times per week, which causes him to have a resting
of theophylline. Because theophylline is metabolized by CYP tremor and feel anxious. One of his colleagues suggests
1A2, 3A3, and 2E1, inhibitors of these isoenzymes can cause clin- that he should inquire about an anticholinergic medication
ically significant interactions.64,99 Cimetidine, clarithromycin, in addition to his ICS. Is this appropriate?
and some (but not all) of the quinolone antibiotics (e.g., enoxacin,
ciprofloxacin) are well documented to inhibit theophylline
metabolism.1,64,99 Because numerous other drugs inhibit the Until now, anticholinergic agents have had limited use in
metabolism of theophylline, all patients receiving this agent asthma except in the ED100 and a few other rare situations.101
P1: Trim: 8.375in × 10.875in Top: 0.373in Gutter: 0.664in
LWBK915-23 LWW-KodaKimble-educational October 27, 2011 8:22

However, in a recent study (n = 210) of tiotropium, a long-acting After reconstitution of omalizumab, the drug must be admin- 593
once-daily anticholinergic agent, in persistent asthmatic patients istered within 4 hours if stored at room temperature and within
poorly controlled on ICS alone, researchers demonstrated that 8 hours when refrigerated. As a result of its viscosity, the injec-
the addition of tiotropium to ICS therapy was not inferior to the tion may take 5 to 10 seconds. No more than 150 mg is injected
effects of adding salmeterol to ICS therapy and was superior to at each site. Although the risk of anaphylaxis is rare (0.2%),111
doubling the inhaled steroid dose.102 The morning and evening patients should stay in the physician’s office for at least 30 min-
PEF were significantly higher (25.8 and 35.3 L/minute, respec- utes after injection of omalizumab and be educated regarding the
tively) in patients receiving tiotropium than those receiving a signs and symptoms of anaphylaxis. After leaving the physician’s
doubling of the ICS dose. office, patients should seek emergency medical treatment at the
Although tiotropium is not identified in EPR-3 as an option first sign of anaphylaxis. Despite the low risk of severe allergic
for persistent asthma patients, a therapeutic trial in R.K. may reactions, in 2010 the manufacturer added a black box warning
be considered given these new data, especially given his anx- about the risk of anaphylaxis.111
iousness and tremor with the increased albuterol use. A similar
response to LABAs may be seen. However, for anticholinergics
to be included in everyday clinical practice, additional long-term EXERCISE-INDUCED ASTHMA

Chapter 23
studies are needed to determine the exact role of tiotropium in
asthma management.
CASE 23-11
QUESTION 1: T.W., a 33-year-old woman, presents to the
Anti-Immunoglobulin E Therapy clinic with a history of severe coughing and chest tight-
ness after exercise. She recently joined an exercise club to
CASE 23-10 lose weight but is unable to keep up with others of her

Asthma
QUESTION 1: M.M. is a 30-year-old woman with severe per- own age and relative condition when jogging outside. She
sistent asthma. Despite optimal assessment, drug therapy, recalls having mild respiratory problems as a young child
environmental control, and patient education per the prin- but has never taken any asthma medications. She has a pos-
ciples of management detailed in EPR-3, she has had two itive treadmill test for exercise-induced asthma (EIA). How
recent hospitalizations as a result of asthma. Her allergist is should T.W. be treated?
considering anti-IgE therapy. Is M.M. a good candidate for
such therapy? How would it be given? M.M. has a pretreat- During sustained exercise, at least 90% of patients with
ment IgE level of 90 international units/mL, and she weighs asthma experience an initial improvement in pulmonary func-
55 kg. tions quickly followed by a significant decline (Fig. 23-12).
This phenomenon may be the only symptom of subclinical
Omalizumab (Xolair) is a humanized monoclonal anti-IgE asthma.1,111 Patients can be diagnosed by measuring the FEV1 or
antibody that binds to free IgE in serum. Thus, binding of IgE PEF before and after exercise (6- to 8-minute treadmill or bicy-
to high-affinity receptors on mast cells is subsequently inhib- cle exercise test). A reduction of FEV1 by more than 15% of the
ited, and the initiation of the allergic inflammatory cascade is baseline value is a positive test.
blocked.1,103–107 Omalizumab is effective in reducing oral and ICS Hyperventilation of cold, dry air increases the sensitivity to
dose requirements in patients with severe asthma and in reducing EIA and induces bronchospasm.111 The main stimulus for EIA
exacerbations.103,104,107–109 This novel therapy is administered as is respiratory heat loss, water loss, or both,111 while breathing
a 150- to 375-mg subcutaneous injection every 2 or 4 weeks. The heated, humidified air completely blocks EIA in many patients.111
dose and frequency of administration are based on the serum Masks are indicated for patients with EIA in the wintertime, and
total IgE level (international units/mL) and the patient’s body patients with severe asthma accompanied by EIA also should
weight. Therefore, M.M.’s omalizumab dose is 150 mg subcuta-
neously every 4 weeks.
Common side effects associated with omalizumab include
injection site reactions, upper respiratory tract infections,
sinusitis, and headache. Less common but potentially serious
adverse effects include anaphylaxis (0.2% in postmarketing
spontaneous reports), which can occur after any dose even
if previous doses have been well tolerated and 24 or more
hours after administration, and the development of malignant
neoplasms (0.5% of omalizumab-treated patients compared
with 0.2% in control patients).
Because omalizumab is expensive and must be administered
as a subcutaneous injection, it should be reserved for patients
with severe asthma who are not adequately controlled with stan-
dard therapies. Despite the high cost, anti-IgE therapy might be
cost-effective in selected patients with severe disease (e.g., those
with frequent ED visits and hospitalizations) because an esti-
mated less than 5% of asthma patients (severe disease) account
for greater than 50% of the dollars spent for asthma care.110

CASE 23-10, QUESTION 2: As the nurse prepares the


dose of omalizumab for M.M., what are some special con- Time (minutes)
siderations regarding administering and monitoring this
drug? FIGURE 23-12 Changes in peak expiratory flow rate with exercise
in an asthmatic and normal subject. PEFR, peak expiratory flow rate.
P1: Trim: 8.375in × 10.875in Top: 0.373in Gutter: 0.664in
LWBK915-23 LWW-KodaKimble-educational October 27, 2011 8:22

594 be encouraged to swim or engage in other indoor exercise that should receive preventive treatment with an inhaled β 2 -agonist.
does not promote EIA. A warm-up period before strenuous exer- The question is whether he should receive a short-acting or a
cise is helpful in some patients. With appropriate premedication, long-acting agent. The clinician should probe as to the duration
most EIA can be prevented, so virtually all patients with sta- of exercise. If W.L. exercises for longer than 3 hours, formoterol or
ble asthma should be encouraged to exercise. The mechanism salmeterol administered 15 to 30 minutes before exercise would
of bronchoconstriction after airway heat and water loss is still be a logical choice. Finally, the clinician should verify that exercise
incompletely understood.111 is the only factor that precipitates asthma symptoms. It could be
Although several drugs inhibit EIA, SABAs are generally the that further questioning of W.L. will reveal persistent asthma or
agents of choice for prophylaxis.1–6,111,112 For typical periods mild intermittent asthma beyond EIA only. If that is the case, long-
of exercise (e.g., <3 hours), pretreatment with agents such as term ICS or montelukast therapy should be started to reduce
albuterol 5 to 15 minutes before exercise usually provides excel- overall airway hyperresponsiveness.
lent protection from EIA.
For prolonged periods of exercise, LABAs (formoterol, sal-
meterol) provide several hours of protection.113 Two differences PATIENT EDUCATION
in formoterol and salmeterol include the delivery systems for
inhalation and the onset of action. If either of these agents is to
Section 3

be used to prevent EIA, it is important for the patient to inhale CASE 23-13
the medication at the proper time before exercise. Formoterol QUESTION 1: A.B., a 26-year-old woman, calls her clinician
should be inhaled at least 15 minutes before exercise, and salme- and states that she has run out of her albuterol MDI. She
terol administration should occur at least 30 minutes before vig- has a prescription for a budesonide dry powder inhaler but
orous activity. Patients who are receiving therapy every 12 hours admits that she does not use this medication “because it
with either drug concomitantly with ICS for long-term control doesn’t seem to work as well as her albuterol MDI.” A.B. has
Pulmonary Disorders

of asthma should already be protected and therefore only use had asthma all of her life. She complains of symptoms most
albuterol if symptoms occur after exercise. With maintenance days but has not required visits to the ED or hospitalizations.
therapy, as opposed to single doses before exercise, bronchopro- The provider determines that A.B. is bothered most about
tection from exercise may be reduced to 5 hours with LABAs.1 daily shortness of breath and worries that her condition may
Leukotriene receptor antagonists (e.g., montelukast once-daily get worse. What should the provider do in this situation?
chronic therapy) have also been demonstrated to prevent EIA.114
Finally, it is important to point out that in persistent asthma, If optimal long-term drug therapy of asthma is prescribed,
long-term anti-inflammatory therapy is helpful in reducing the treatment may still fail or be suboptimal if the patient does not
response to most asthma triggers, including exercise.1 For most receive adequate education. Patients with asthma require special
patients who have EIA only, use of a SABA 15 minutes before educational efforts because of the use of inhalation devices and
exercise is the only therapy needed. peak flow meters. In addition, it often is a major challenge to
Because of the hyperventilation of relatively cool, dry air, have patients and parents understand the critical importance of
jogging is a potent stimulus for EIA. A number of possible ther- long-term daily controller therapy and environmental control.
apeutic interventions exist for T.W. She could be encouraged to Of course, an important first step in educating asthma patients is
swim because the inhalation of humidified warm air is less likely to be caring and a good listener. Rather than sharing your knowl-
to produce EIA. However, if she wishes to continue jogging, edge initially, it is important to help establish a “partnership”
two inhalations of a SABA (e.g., albuterol) from a metered-dose with the patient by first asking the following question: “What
aerosol 15 minutes before exercise should provide adequate pro- is bothering you the most about your asthma?” Really listening
tection for 2 to 3 hours. If outdoor temperatures are quite cool or to the patient and then addressing patient concerns is extremely
cold, T.W. should jog indoors. T.W. also should be counseled to important to successful education and long-term management.
take two additional inhalations if she “breaks through” the initial EPR-3 lists several patient education activities in Table 23-13.1
protection and experiences tightness. Clinicians can be of invaluable assistance to the patient by
repeatedly reinforcing education on the necessity of using anti-
CASE 23-12 inflammatory (and combined ICS/LABA) therapies on a reg-
ular schedule. Many patients underuse long-term preventive
QUESTION 1: W.L., a 17-year-old boy, presents to the clinic
therapy because no health professional took the time to ade-
with a complaint of dyspnea and coughing that has limited
quately instruct them that most asthma symptoms are pre-
his ability to keep up with his basketball teammates. He
ventable. Although underusing the most important medicines
states that it is worse when playing outdoors unless the
for long-term control, many patients overuse “quick relievers”
gym is cold and that it seems to be worse (occurring sooner
(i.e., SABAs). Health care providers must be able to detect these
during exercise) than a month ago. W.L. experienced several
problems and intervene to enhance patient care.
bouts of bronchitis as a young child but has not had any
Because a large percentage of patients have difficulty using
problems for the past 6 years. His symptoms are consistent
MDIs, teaching patients the correct use of MDIs (alone or in
with EIA. How should his EIA be treated?
combination with spacers) and dry powder inhalers is absolutely
essential.1,115 In one study, 89% of patients could not perform
W.L. presents a special problem in that he is a teenager. Both all of the steps for MDI use correctly.1,116 Competent teaching
for adolescents and children, peer pressure usually is extremely requires observation of the patient using the devices initially
significant. Optimal prophylaxis is important to allow W.L. to and again on repeat visits to the clinic, hospital, or community
compete at his best level. Embarrassment about not keeping up pharmacy. Telling the patient about correct use is inadequate.
with teammates can be very hurtful now, and it has implications Health professionals must demonstrate use of the devices (live
for setting habits of exercise into adulthood. Many adults with or with videotapes) for patients who cannot use the devices
asthma do not exercise because they think they cannot do so correctly. For videos that show proper use of many types
based on childhood experiences. Lack of exercise can have a of asthma inhalers, go to http://www.nationaljewish.org/
negative impact on physiologic and psychologic well-being. W.L. healthinfo/multimedia/asthma-inhalers.aspx. for a video of
P1: Trim: 8.375in × 10.875in Top: 0.373in Gutter: 0.664in
LWBK915-23 LWW-KodaKimble-educational October 27, 2011 8:22

595
TA B L E 2 3 - 1 3 TA B L E 2 3 - 1 4
Key Educational Messages: Teach and Reinforce at Steps to Correct Use of Metered-Dose Inhalersa
Every Opportunity
1. Shake the inhaler well and remove the dust cap.
Basic Facts About Asthma 2. Exhale slowly through pursed lips.b
s
The contrast between airways of a person who has and a person 3. If using the “closed-mouth” technique, hold the inhaler upright and
who does not have asthma; the role of inflammation place the mouthpiece between your lips. Be careful not to block the
s
What happens to the airways in an asthma attack opening with your tongue or teeth.
Roles of Medications—Understanding the Difference Between the 4. If using the “open-mouth” technique, open your mouth wide and
Following: hold the inhaler upright 1–2 inches from your mouth, making sure
s
Long-term-control medications: prevent symptoms, often by the inhaler is properly aimed.
reducing inflammation. Must be taken daily. Do not expect them 5. Press down on the inhaler once as you start a slow, deep inhalation.
to give quick relief. 6. Continue to inhale slowly and deeply through your mouth. Try to
s
Quick-relief medications: short-acting β 2 -agonists relax muscles inhale for at least 5 seconds.
around the airway and provide prompt relief of symptoms. Do not 7. Hold your breath for 10 seconds (use your fingers to count to 10
expect them to provide long-term asthma control. Using slowly). If 10 seconds makes you feel uncomfortable, try to hold
your breath for at least 4 seconds.

Chapter 23
quick-relief medication on a daily basis indicates the need for
starting or increasing long-term control medications. 8. Exhale slowly.c
9. Wait at least 30–60 seconds before inhaling the next puff of medicine.
Patient Skills
s
Taking medications correctly a
If using a spacer, see manufacturer’s instructions. Same basic principles of slow,
– Inhaler technique (demonstrate to patient and have the patient deep inhalation with adequate breath-hold apply. With spacers, put mouthpiece
return the demonstration) on top of your tongue to ensure that tongue does not block aerosol.
b
– Use of devices, such as prescribed valved holding chamber, As long as exhalation is slow, exhale can take place for several seconds. Some

Asthma
spacer, nebulizer experts insist on exhaling only a tidal volume, but the key is to exhale slowly.
c
s
Identifying and avoiding environmental exposures that worsen the If patient has concomitant rhinitis, exhaling through the nose may be of benefit
when using corticosteroids or ipratropium (i.e., some medication may deposit in
patient’s asthma (e.g., allergens, irritants, tobacco smoke)
s nose).
Self-monitoring to:
– Assess level of asthma control
– Monitor symptoms and, if prescribed, peak flow
– Recognize early signs and symptoms of worsening asthma Such rapid PIF is achievable by some young children, but many
s
Using written asthma action plan to know when and how to: children younger than 8 years of age have difficulty reaching PIF
– Take daily actions to control asthma greater than 60 L/minute.126 With the Diskus, PIF does not have
– Adjust medication in response to signs of worsening asthma to be as rapid as with Flexhaler, but it should be greater than
– Seek medical care as appropriate 30 L/minute.127 In addition, patients need to breath-hold 10 sec-
onds if possible, as with an MDI.
Reprinted from National Institutes of Health. Expert Panel Report 3: Guidelines for
the Diagnosis and Management of Asthma. Bethesda, MD: National Heart, Lung,
and Blood Institute; 2007. NIH publication 07-4051.
Asthma Self-Management Plans
Objective monitoring of lung function at home with the use
how to use a nebulizer, go to http://www.nationaljewish.org/ of peak flow meters can be very helpful to patients and health
healthinfo/medications/lung-diseases/devices/nebulizers/ care professionals. EPR-3 discusses the debates about PEF ver-
instructions.aspx. sus symptom-based action plans.1 Use of peak flow meters may
Although there is more than one correct way to use be valuable in patients who have had severe exacerbations and
an MDI, Table 23-14 summarizes two commonly accepted those who are “poor perceivers” of deteriorating asthma control.
approaches.1,115 Many asthma experts prefer to use spacers to Instructing patients on the correct use of the devices, including
help ensure optimal efficacy. Spacers should be used in virtually use of the green, yellow, and red zones, is essential.1 After estab-
all patients receiving ICS via an MDI, even those with perfect lishing that optimal therapy has maintained the PEF in the “green
MDI technique, because spacers enhance efficacy and greatly zone” in the early morning, most patients can simply verify their
reduce the risk of oropharyngeal candidiasis.117–119 On the other values once daily in the early morning. Analogous to a traffic
hand, spacers do not add efficacy to correct use of a β 2 -agonist light, green, yellow, and red zones have been established to guide
MDI.120 Although any spacer can be helpful, marketed devices the patient and clinician. The green zone refers to a PEF that is 80%
that have a flow indicator whistle when inhalation is fast may be to 100% of “personal best” and generally indicates that therapy
preferred (e.g., AeroChamber). is providing good control. Before a course of optimal therapy
Studies have shown that health professionals, like some to attain a personal best, the zones are set based on predicted
patients, generally are not competent in using MDIs.121–123 Obvi- values found in each peak flow meter package insert. The yellow
ously, the clinician should practice with a placebo inhaler and zone indicates a PEF that is 50% to 79% of personal best. Patients
gain competence before teaching a patient. Among clinicians should be instructed to call their physician or other health care
who educate asthmatic patients, pharmacists can be very help- provider for adjustment in preventive medication if the PEF stays
ful in teaching correct use of MDIs.124 Unfortunately, one study in the yellow zone after using two puffs of a β 2 -agonist. The red
showed that community pharmacists commonly are not provid- zone indicates a PEF that is less than 50% of personal best. The
ing such teaching.125 patient should know to call his or her health care provider imme-
In addition to teaching the correct use of MDIs and spacers, diately if the use of an inhaled β 2 -agonist does not bring the PEF
clinicians should help patients via education regarding the cor- to the yellow or green zone. Figure 23-13 gives an example of
rect use of breath-activated dry powder inhalers (e.g., Twisthaler, PEF monitoring, and Figure 23-14 gives an example of a written
Diskus, Aerolizer), breath-activated MDIs (e.g., Autohaler), and action plan.
nebulizing machines.1 When using the Flexhaler, for example, Correct use of the peak flow meter includes standing, inhal-
patients must clearly understand the need for a rapid (prefer- ing completely, forming a tight seal with the lips around the
ably 60 L/minute), deep inhalation (not slow as with an MDI).124 mouthpiece, exhaling as hard and fast as possible (blast!), and
P1: Trim: 8.375in × 10.875in Top: 0.373in Gutter: 0.664in
LWBK915-23 LWW-KodaKimble-educational October 27, 2011 8:22

596 sprayed onto the face or into the eyes. As the open-mouth tech-
Janie Doe Height 49” Patient’s Personal Best 280
nique has not been studied with the currently available MDIs,
Date 4/22 4/23 4/24 4/25 4/26 4/27 4/28 4/29 4/30 5/1 5/2 5/3 5/4
it is recommended that the closed-mouth technique be used as
300
the nonchlorofluorocarbon MDIs were FDA approved using this
method. Finally, regarding the HFA MDIs, special care should be
followed in cleaning the actuators.131 Health care professionals
PEF (L/min)

200 Visited
friend
with
cat Beta
Upper
respiratory
should stress the importance of regular MDI actuator cleaning
100
agonist infection with their patients.
Burst
of oral
steroids
CASE 23-13, QUESTION 3: For patients who are using both
a bronchodilator and an anti-inflammatory inhaler, is there
FIGURE 23-13 Asthma changes with time: patient monitoring and a preferred sequencing of the inhalers?
follow-up required. PEF, peak expiratory flow. Adapted from
National Institutes of Health. Practical Guide for the Diagnosis and For patients who have several inhalers, questions regarding
Management of Asthma. Bethesda, MD: National Heart, Lung, and sequencing of the inhalers are frequently asked. First, there is no
Blood Institute; 1997. NIH publication 97-4053.
well-documented evidence that outcomes are better using, for
Section 3

instance, a bronchodilator or an anti-inflammatory agent first. A


repeating this maneuver twice (for a video that shows correct commonsense approach is that using a rapid-onset bronchodila-
use of a peak flow meter, see http://www.monaghanmed. tor such as a β 2 -agonist first and then an anti-inflammatory sec-
com/products/consumer/truzone-peak-flow-meter-pfm). ond has some appeal (i.e., quick relief and theoretically enhanced
The best of three attempts should be recorded. Beyond giving penetration of the anti-inflammatory). However, as discussed
maximal effort when using peak flow meters, patients should be previously, SABAs are preferred for as-needed use (and before
Pulmonary Disorders

instructed to place the instrument well into the mouth on top exercise) and are not generally used on a scheduled basis. Thus,
of the tongue to avoid acceleration of air in the mouth with the if a patient is not symptomatic at the time the anti-inflammatory
tongue and buccal musculature. In essence, “spitting” into the is scheduled, current literature suggests that the patient inhales
peak flow meter causes a dramatic false elevation in PEF.128 Some only the anti-inflammatory agent. Therefore, it is usually not
data suggest that women need more initial coaching than men necessary to counsel patients regarding sequencing of inhaled
in giving maximal effort when using peak flow meters to ensure medications. Because time is limited in counseling patients and,
accurate assessments of PEF.129 teaching them correct inhalation technique, the purpose of each
A.B. needs education regarding the benefits of long-term medication (controllers vs. quick relievers), and the need for strict
inhaled anti-inflammatory therapy. The clinician should explain adherence with controller therapies is far more important than
with enthusiasm that A.B.’s budesonide is an extremely effective spending precious time on the sequencing of inhalers.
medicine and that it is the cornerstone of her asthma manage-
ment. The delayed onset and safety of ICS must be stressed as well
as the requirement of regular use every day. Teaching A.B. the NOCTURNAL ASTHMA
differences between preventers and quick relievers is essential.
Showing her colored pictures, models, or a video of inflamed
airways can be very helpful—these teaching aids are available CASE 23-14
from several pharmaceutical manufacturers. Likewise, a peak QUESTION 1: R.R., a 41-year-old man, presents to the clinic
flow meter should be given to A.B. and correct use ensured by with a history of coughing and shortness of breath that
observing her use of it along with establishment of green, yellow, awakens him at least two nights a week. Most mornings
and red zones, coupled with a written action plan developed by on awakening, he complains of chest tightness. He has a
her asthma care provider.1 A.B. needs to hear from the clinicians history of asthma since childhood and currently is managed
treating her that asthma is preventable, and in the words of a title with beclomethasone HFA 160 mcg BID via a spacer and
to an NIH booklet for patients, “Your asthma can be controlled: albuterol (90 mcg/puff) two puffs every 6 hours PRN and
Expect nothing less.” As part of comprehensive education, such before exercise. R.R.’s morning PEF is consistently in the
a positive message from all of her caregivers, as well as carefully yellow zone, usually at about 400 L /minute (personal best,
listening to A.B.’s concerns, can have a major impact on A.B., 600 L /minute), whereas the evening PEF is consistently
who has not been managing her asthma optimally. 550 to 600 L /minute. What treatment should be recom-
mended?
CASE 23-13, QUESTION 2: A.B. tells the provider that she
was previously instructed to place the albuterol MDI in front Many patients with asthma complain of symptoms that
of her open mouth and spray rather than put the MDI in her awaken them in the night or occur on awakening in the morn-
mouth. She says she is confused because the package insert ing. Morning cough with or without bronchospasm may be a
shows placement of the inhaler in the mouth. What should clue to nocturnal asthma. Although nocturnal asthma may be
the clinician tell A.B.? appropriately viewed as simply another manifestation of airway
inflammation, it is so common and troublesome among asth-
A.B. is correct that this is a confusing issue to many patients matic patients that it deserves special note. Circadian rhythm in
and health professionals. A small number of studies show that PEF is exaggerated in patients with asthma. The difference in PEF
the “open-mouth” technique is better, but several other stud- in nonasthmatic patients averages about 8% between 4 pm (max-
ies with SABAs show that the “closed-mouth” technique is as imal airflow) and 4 am (minimal airflow), but in patients with
good as or better than putting the MDI in front of the open asthma, the average variation can be as high as about 50%.132,133
mouth.1,115 In addition, the correctly performed closed-mouth Several mechanisms account for this diurnal variation in PEF.
technique is as efficacious with a β 2 -agonist as with a spacer120 The following are examples of factors that contribute to noctur-
or nebulizer.130 One caution with the open-mouth technique is nal asthma: increased release of inflammatory mediators,132,133
that incorrect aiming of the MDI may result in the aerosol being increased activity of the parasympathetic nervous system, lower
P1:
Asthma Action Plan LWBK915-23

For: Doctor: Date:


Trim: 8.375in × 10.875in

Doctor’s Phone Number Hospital/Emergency Department Phone Number

Doing Well Take these long-term control medicines each day (include an anti-inflammatory).
Medicine How much to take When to take it
No cough, wheeze, chest tightness, or
shortness of breath during the day or night
Can do usual activities
Top: 0.373in
LWW-KodaKimble-educational

GREEN ZONE
And, if a peak flowmeter is used,

Peak flow: more than Identify and avoid and control the things that make your asthma worse, like (list here):
(80% or more of my best peak flow)

My best peak flow is:


Gutter: 0.664in

Before exercise, if prescribed, take: 2 or 4 puffs 5 to 60 minutes before exercise

First Add: quick-relief medicine—and keep taking your GREEN ZONE medicine.
Asthma Is Getting Worse
Cough, wheeze, chest tightness, or 2 or 4 puffs, every 20 minutes for up to 1 hour
shortness of breath, or (short-acting β 2-agonist) Nebulizer, once
Waking at night because of asthma, or Second If your symptoms (and peak flow, if used) return to GREEN ZONE after 1 hour of above treatment:
Can do some, but not all, usual activities Continue monitoring to be sure you stay in the Green Zone.

YELLOW ZONE
-Or- -Or-
If your symptoms (and peak flow, if used) do not return to GREEN ZONE after 1 hour of above treatment:
Peak flow: to Take: 2 or 4 puffs or Nebulizer
(50% to 79% of my best peak flow) (short-acting β 2-agonist)
Add: mg per day For (3–10) days
(oral corticosteroid)
Call the doctor before/ within hours after taking the oral steroid.

Medical Alert! Take this medicine:

Very short of breath, or 4 or 6 puffs or Nebulizer


Quick-relief medicines have not helped, or (short-acting β 2-agonist)

RED ZONE
Cannot do usual activities, or mg
Symptoms are same or get worse after (oral corticosteroid)
24 hours in Yellow Zone
Then call your doctor NOW. Go to the hospital or call an ambulance if:
-Or- You are still in the Red Zone after 15 minutes AND
You have not reached your doctor.
Peak flow: less than
(50% of my best peak flow)

DANGER SIGNS Trouble walking and talking because of shortness of breath Take 4 or 6 puffs of your quick-relief medicine AND
Lips or fingernails are blue Go to the hospital or call for an ambulance NOW!
(phone)

FIGURE 23-14 Sample asthma action plan. (Adapted from National Institutes of Health. Expert Panel Report 3. Guidelines for the Diagnosis and Management of
Asthma. NIH Publication No. 08-4051.0; 2007.
October 27, 2011

Asthma Chapter 23
597
8:22
P1: Trim: 8.375in × 10.875in Top: 0.373in Gutter: 0.664in
LWBK915-23 LWW-KodaKimble-educational October 27, 2011 8:22

598 circulating levels of epinephrine, and lower levels of serum cor- inal agents has been reported numerous times. The most exten-
tisol (lowest at about midnight). In addition, for patients whose sive literature on drug-induced asthma involves NSAIDs and
asthma is triggered by gastroesophageal reflux, this problem is β-blockers. Other drugs and drug preservatives also can induce
worse at night and is another factor to consider. symptoms of asthma, but because the topic is beyond the scope
The initial approach to managing nocturnal symptoms is the of this chapter, the reader is referred to other sources136,137 for
same as that for overall long-term therapy of persistent asthma, further discussion.
including adequate anti-inflammatory agents.1,133 ICS are often The percentage of asthmatic patients reported to be aspirin
effective in eliminating or reducing nocturnal asthma, including sensitive ranges from 4% to 28%. Clinical manifestations of
symptoms and the drop in PEF.133 If low to moderate dosages aspirin sensitivity include rhinorrhea, mild wheezing, or severe,
(i.e., correctly inhaled every day) do not eliminate symptoms, life-threatening shortness of breath. Once the reaction has
a LABA (salmeterol, formoterol) is indicated. Also, the basic occurred, there is a refractory period of 2 to 5 days.138 If an asth-
asthma treatment principle of good control of concomitant rhini- matic patient is aspirin sensitive, it is likely that the patient also
tis and environmental control, especially in the bedroom (e.g., will react to most other NSAIDs. Aspirin and other NSAIDs share
house dust mites, household pets), should be considered in the common mechanisms involving the arachidonic acid pathways,
patient with nocturnal asthma symptoms. including inhibition of cyclo-oxygenase, which results in more
Bedtime doses of SABAs do not have sufficient duration of rapid synthesis and overproduction of leukotrienes.138 Not sur-
Section 3

action to prevent early morning symptoms. Salmeterol or for- prisingly, because leukotrienes are an important part of the mech-
moterol, both of which have a 12-hour duration of action, are anism of NSAID-induced asthma, inhibitors of 5-lipoxygenase
preferred (i.e., added to ICS). such as zileuton are generally, but not always, effective in blocking
Before the advent of LABAs, long-acting oral agents such as SR this response.139 Similarly, leukotriene receptor antagonists such
theophylline often were indicated.133 Although SR theophylline as zafirlukast and montelukast are generally, but not always, effec-
is helpful, it has the potential to cause more adverse effects than tive in blocking aspirin-induced asthma.140 Because most patients
Pulmonary Disorders

inhaled agents, may interfere with sleep, and is less efficacious.133 with asthma do not react to aspirin and other NSAIDs, the NIH
Before a long-acting oral agent is prescribed, adequate inhaled guidelines recommend avoiding these agents only in patients
anti-inflammatory therapy should be ensured and then LABAs with known sensitivity.1 In addition, patients with severe persis-
should be added. tent asthma or nasal polyps should be counseled regarding the
Because asthma is primarily an inflammatory disease and noc- risks associated with these drugs. In patients with known sensitiv-
turnal symptoms are largely caused by airway inflammation, ity, acetaminophen or salsalate are recommended for headaches
the first drug therapy concern in R.R. is to ensure that he is and relatively minor pain.1 For patients who are sensitive but who
strictly adhering to his beclomethasone therapy and demonstrat- need to take aspirin (e.g., after myocardial infarction [MI]) or an
ing excellent inhalation technique. If his use of the medication is NSAID (e.g., arthritis), it is possible to desensitize the patient,
optimal, a reasonable approach would be to add LABA therapy, and daily use then prevents further reaction.134
because he is already at a moderate ICS dosage. When discussing drug-induced asthma, the other major con-
As part of optimal management of nocturnal asthma, R.R. sideration is β-blockers. These agents should be used with great
also should be asked about avoiding or minimizing exposure to caution in patients with asthma. Because even β 1 -adrenergic
his asthma triggers (e.g., if he is allergic to cats, is there a cat in blockers lose selectivity as dosages are increased, they, as well as
the bedroom?). Follow-up visits for R.R. should verify that the nonselective β-blockers, should be avoided in most patients. Fur-
early morning and evening PEF are staying in the green zone thermore, ophthalmic timolol has been reported several times to
and that symptoms, both during the night and on awakening in cause fatal asthma and should absolutely be avoided in patients
the morning, have been eliminated. with a history of asthma.141 Other β-blocker eye drops (e.g.,
betaxolol) have been reported to have less propensity to induce
asthma, but all have some risk.136,142
DRUG-INDUCED ASTHMA Two notable exceptions to using β-blockers in patients with
asthma are patients who are post-MI and patients with heart
failure.143 Because β-blockers prolong life after an MI and
CASE 23-15 improve the care of patients with heart failure, benefits versus
QUESTION 1: M.B., a 32-year-old woman with asthma, asks risks should be weighed. Risks outweigh benefits if a patient
her clinician which over-the-counter medications would be has severe persistent asthma.143,144 If a post-MI patient has mild
preferred for minor aches and pains. M.B. says that she is intermittent asthma or well-controlled mild persistent (and pos-
very sensitive to aspirin (causes severe wheezing). sibly moderate persistent)143 asthma with optimal management,
a low dosage of atenolol 50 mg/day is a reasonable consideration
The clinician should counsel the patient regarding the fact that in which benefits may outweigh risks.144 Asthma patients have
patients with asthma who are aspirin sensitive often react to other been shown to respond to inhaled β 2 -agonists when receiving
nonsteroidal anti-inflammatory drugs (NSAIDs) such as ibupro- this dosage of atenolol.144 Although low dosages of β-blockers
fen by developing asthma symptoms with the first dose. The are not proven to prolong life after an MI, some studies sug-
health care professional should suggest acetaminophen. If M.B. gest efficacy of lower dosages. For patients with heart failure,
says that acetaminophen does not give adequate relief of her pain, metoprolol CR/XL is the cardioselective β-blocker approved in
other options are salsalate2 or consultation with a board-certified the United States, whereas carvedilol, which has nonselective
allergist with experience in aspirin and NSAID desensitization.134 β-blocker as well as α-blocking properties, can worsen asthma
The clinician should also suggest consultation with an aller- symptoms.144
gist regarding a recent study that gives strong evidence that If a patient with asthma is given a β-blocker and ini-
cyclo-oxygenase-2 (COX-2) inhibitors are safe in aspirin-sensitive tially reports no symptoms, subsequent exacerbations may not
asthma.135 This case also points out the need for health profes- respond well to administration of usual doses of a β-agonist.
sionals to pay attention to patient use of nonprescription medi- The drug of choice for β-blocker–induced bronchospasm is
cations. Although drug-induced asthma may present as relatively ipratropium.144 A more subtle risk with β-blockers involves the
mild symptoms in some patients, fatal asthma caused by medic- adult with allergic rhinitis and a family history of asthma. If this
P1: Trim: 8.375in × 10.875in Top: 0.373in Gutter: 0.664in
LWBK915-23 LWW-KodaKimble-educational October 27, 2011 8:22

individual is given a β-blocker for hypertension, symptoms of EPR-31 ) as well as reduction in ED visits and hospitalizations.1 599
asthma could be induced, especially if another trigger is intro- To achieve optimal outcomes, attention to each of the four major
duced such as running in cold, dry air. components of management is required (objective assessment,
environmental control, pharmacologic therapy, and patient edu-
cation as a partnership). Examples of areas that pose special chal-
OUTCOMES lenges for inner-city patients include psychosocial factors, under-
use of controller medications, and passive cigarette smoke.153–155
As part of overall management to improve outcomes, recent
CASE 23-16 studies have emphasized again the importance of good inhala-
QUESTION 1: C.C. is a 36-year-old woman admitted to the tion technique with ICS and education in the use of peak flow
hospital for asthma. This is her second hospitalization in meters.156,157
the past 2 years, and she has had three ED visits during
the same period. She also complains of nocturnal awaken-
ings at least 4 nights per week and is bothered that she is CASE 23-16, QUESTION 2: C.C. returns to the clinic in 2
gaining weight because she cannot exercise. Lack of exer- months and is elated because she is sleeping through the

Chapter 23
cise is also troubling her because her 5-year-old daughter night and not waking up with shortness of breath. In addi-
wants her to go outside and play with her. C.C. has been tion, she is beginning to exercise again, which makes her
inhaling fluticasone 44 mcg one puff BID without a spacer and her child very happy. C.C. has had no further ED visits.
for years along with frequent PRN albuterol. C.C. carefully What should the clinician do at this point?
controls her home environment. What could her clinicians
do to improve her outcomes, including quality of life? Optimal asthma management that improves outcomes is a
continuous process of education and reassessment of the overall

Asthma
C.C. needs a reassessment of her long-term management therapy. Observation of C.C. using her peak flow meter and
considering her very poor outcomes during the past 2 years. inhalation devices on each clinic visit is important and should
First, her clinicians need to establish a partnership with her be routine. Having C.C. verbalize her understanding of the role
in education regarding asthma and its management. Clearly, of the ICS plus LABA versus albuterol and the action plan with
she needs a dosage adjustment with her anti-inflammatory “crisis” prednisone is important. Despite her current optimism,
therapy because her dosage is too low and she is experienc- asking C.C. about her current asthma concerns is important.
ing symptoms of poor asthma control. Given her recent his- During the next month or two, a trial of slowly stepping down
tory, she should be treated initially with high-dose ICS and a the ICS dosage to a medium dose should be attempted. Finally,
LABA, preferably with a more potent ICS product that would C.C. needs continued partnership with her clinician.
require fewer inhalations per day (e.g., combination budesonide-
formoterol, combination mometasone-formoterol, or combina-
tion fluticasone-salmeterol). C.C. should have the importance CASE 23-16, QUESTION 3: C.C. is in the clinic 2 years later,
of daily controller therapy stressed, including the need for strict reflecting with her clinician about her total elimination of ED
adherence and proper inhalation technique. C.C. needs a writ- visits and hospitalizations as well as her improved quality of
ten asthma action plan and an emergency supply of prednisone life. Her management initiated 24 months ago has contin-
(i.e., to use when her PEF is in the red zone and unresponsive ued, including environmental control, controller therapy tai-
to albuterol). C.C. should be told to expect a reduced need for lored for her, PRN albuterol, early morning PEF monitoring,
albuterol. and partnership with her clinician. Unfortunately, C.C. for-
Numerous studies have documented that applying the princi- got to get an influenza vaccine last October and became ill
ples of the NIH guidelines1–6 results in improved clinical out- with influenza in early March. Although this episode only
comes, and many of these studies have been summarized in slightly worsened her asthma symptoms, when she was
EPR-3.1 Several studies have documented that pharmacists who almost recovered from the flu, she went to the grocery store
are highly knowledgeable of the NIH guidelines and who work and breathed secondhand smoke unexpectedly. In addition,
closely with patients and physicians improve outcomes, includ- early spring tree and grass pollen was affecting her aller-
ing reduced ED visits and hospitalizations.145–150 These success- gic rhinitis. By the time she got back to her house, she
ful studies involved highly motivated pharmacists, who were was wheezing and her PEF was in the yellow zone, but it
asthma experts based in university-affiliated clinics or in large responded to three puffs of albuterol. C.C. asks what she
private health maintenance organizations. should have done if this series of events had resulted in her
A recent randomized controlled trial based in community PEF decreasing to the red zone.
pharmacies with specially trained pharmacists has also shown
very positive outcomes for patients with asthma.151 Another C.C. needs to be re-educated regarding the action plan based
randomized controlled trial based in chain drug stores with on symptoms and PEF values. Referring back to the written plan
staff pharmacists did not show a benefit related to attempts at for doses of albuterol and, if needed, oral corticosteroid therapy
asthma care.152 Unfortunately, the level of training and incen- is important. Emphasizing the need for annual influenza vaccina-
tives did not appear optimal, and the authors pointed out that tion in October is important. Reinforcement of the importance
the staff pharmacists were “not universally enthusiastic” about of continued preventive therapy that has given such remarkable
the program. The authors also described their intervention as success is appropriate for C.C. and reassuring her that despite
“cumbersome” for the pharmacists. Further study is needed to this minor setback, she is in control of her asthma. The clinician
assess asthma care in this setting when pharmacists are optimally should continue to work with her to further tailor the therapy,
trained, given appropriate incentives, and enthusiastic about the including control of rhinitis, to maintain optimal outcomes at
program. the lowest dosages and the simplest possible regimen. Recent
When assessing the effect of comprehensive management on research has further emphasized the goal of simplified regimens
clinical outcomes, quality-of-life measures should be assessed (a and achieving the lowest effective doses of anti-inflammatory
validated questionnaire (the Asthma Control Test) is shown in therapy.158,159
P1: Trim: 8.375in × 10.875in Top: 0.373in Gutter: 0.664in
LWBK915-23 LWW-KodaKimble-educational November 22, 2011 15:25

600 COMPLEMENTARY ALTERNATIVE Dolovich MB et al. Device selection and outcomes of aerosol
THERAPIES therapy: evidence-based guidelines: American College of Chest
Physicians/American College of Asthma, Allergy, and Immunol-
ogy. Chest. 2005;121:335. (28)
CASE 23-16, QUESTION 4: C.C. is in the clinic a few months
Holgate ST, Polosa R. The mechanisms, diagnosis, and manage-
later. She is continuing to have excellent control of her
ment of severe asthma in adults. Lancet. 2006;368:780. (12)
asthma and allergic rhinitis. C.C. asks her health care pro-
fessional for her opinion of herbal remedies for asthma as Lemanske RF Jr, Busse WW. The US Food and Drug Administra-
well as other nontraditional approaches to treatment. tion and long-acting beta2 -agonists: the importance of striking
the right balance between risks and benefits of therapy. J Allergy
Complementary and alternative approaches that have been Clin Immunol. 2010;126:449. (94)
used in the treatment of asthma include black tea, coffee, ephedra, Newman SP. Spacer devices for metered dose inhalers. Clin Phar-
marijuana, dried ivy leaf extract, acupuncture, meditation, and macokinet. 2004;43:349. (119)
yoga.1,160,161 Despite the widespread use of alternative medica-
tions for chronic conditions, the clinician should discuss with Self TH et al. Reducing emergency department visits and hos-
C.C. that there is no established scientific basis for their use in the pitalizations in African Americans and Hispanic patients with
Section 3

management of asthma.1 Complementary alternative medicine asthma: a 15-year review. J Asthma. 2005;42:807. (155)
cannot be recommended as a substitute for the drug therapy Suissa S et al. Low-dose inhaled corticosteroids and the preven-
recommended by EPR-3 and other medical literature based on tion of death from asthma. N Engl J Med. 2000;343:332. (85)
randomized controlled studies.
Tan RA, Spector SL. Exercise-induced asthma: diagnosis
and management. Ann Allergy Asthma Immunol. 2002;89:226.
KEY REFERENCES AND WEBSITES
Pulmonary Disorders

(112)
A full list of references for this chapter can be found at
http://thepoint.lww.com/AT10e. Below are the key references Key Websites
and websites for this chapter, with the corresponding reference Global Initiative for Asthma (GINA) 2010 Update. http://
number in this chapter found in parentheses after the reference. ginasthma.com/Guidelineitem.asp??l1=2&l2=1&intId=60.
National Institutes of Health. Expert Panel Report 3. Guidelines
Key References for the Diagnosis and Management of Asthma. Bethesda, MD:
Bateman ED et al. Can guideline defined asthma control be National Heart, Lung, and Blood Institute; 2007. NIH publica-
achieved? The Gaining Optimal Asthma Control Study. Am J tion 07-4051. http://www.nhlbi.nih.gov/guidelines/asthma/
Respir Crit Care Med. 2004;170:836. (90) asthgdln.pdf. (1)

Das könnte Ihnen auch gefallen